Примеры решение системы уравнений методом гаусса: Решение систем линейных уравнений методом Гаусса

Содержание

принцип, теорема и примеры решения задач

Задание. Решить СЛАУ
$\left\{\begin{array}{l}
2 x_{1}+x_{2}+x_{3}=2 \\
x_{1}-x_{2}=-2 \\
3 x_{1}-x_{2}+2 x_{3}=2
\end{array}\right.$ методом Гаусса.

Решение. Выпишем расширенную матрицу системы и при помощи элементарных преобразований над ее
строками приведем эту матрицу к ступенчатому виду (прямой ход) и далее выполним обратный ход метода Гаусса
(сделаем нули выше главной диагонали). Вначале поменяем первую и вторую строку, чтобы элемент $a_{11}$ равнялся 1 (это мы делаем для упрощения
вычислений):

$$\tilde{A}=A \mid B=\left(\begin{array}{rrr|r}
2 & 1 & 1 & 2 \\
1 & -1 & 0 & -2 \\
3 & -1 & 2 & 2
\end{array}\right) \sim\left(\begin{array}{rrr|r}
1 & -1 & 0 & -2 \\
2 & 1 & 1 & 2 \\
3 & -1 & 2 & 2
\end{array}\right)$$

Далее делаем нули под главной диагональю в первом столбце. Для этого от второй строки отнимаем две первых,
от третьей — три первых:

$$\tilde{A} \sim\left(\begin{array}{rrr|r}
1 & -1 & 0 & -2 \\
0 & 3 & 1 & 6 \\
0 & 2 & 2 & 8
\end{array}\right)$$

Все элементы третьей строки делим на два (или, что тоже самое, умножаем на $\frac{1}{2}$ ):

$$\tilde{A} \sim\left(\begin{array}{rrr|r}
1 & -1 & 0 & -2 \\
0 & 3 & 1 & 6 \\
0 & 1 & 1 & 4
\end{array}\right)$$

Далее делаем нули во втором столбце под главной диагональю, для удобства вычислений
поменяем местами вторую и третью строки, чтобы диагональный элемент равнялся 1:

$$\tilde{A} \sim\left(\begin{array}{ccc|c}
1 & -1 & 0 & -2 \\
0 & 1 & 1 & 4 \\
0 & 3 & 1 & 6
\end{array}\right)$$

От третьей строки отнимаем вторую, умноженную на 3:

$$\tilde{A} \sim\left(\begin{array}{rrr|r}
1 & -1 & 0 & -2 \\
0 & 1 & 1 & 4 \\
0 & 0 & -2 & -6
\end{array}\right)$$

Умножив третью строку на $\left(-\frac{1}{2}\right)$ , получаем:

$$\tilde{A} \sim\left(\begin{array}{rrr|r}
1 & -1 & 0 & -2 \\
0 & 1 & 1 & 4 \\
0 & 0 & 1 & 3
\end{array}\right)$$

Проведем теперь обратный ход метода Гаусса (метод Гассу-Жордана), то есть сделаем нули над главной диагональю.
Начнем с элементов третьего столбца. Надо обнулить элемент $a_{23}$, для этого от второй строки отнимем третью:

$$\tilde{A} \sim\left(\begin{array}{rrr|r}
1 & -1 & 0 & -2 \\
0 & 1 & 0 & 1 \\
0 & 0 & 1 & 3
\end{array}\right)$$

Далее обнуляем недиагональные элементы второго столбца, к первой строке прибавляем вторую:

$$\tilde{A} \sim\left(\begin{array}{ccc|c}
1 & 0 & 0 & -1 \\
0 & 1 & 0 & 1 \\
0 & 0 & 1 & 3
\end{array}\right)$$

Полученной матрице соответствует система

$\left\{\begin{array}{l}x_{1}+0 \cdot x_{2}+0 \cdot x_{3}=-1 \\ 0 \cdot x_{1}+x_{2}+0 \cdot x_{3}=1 \\ 0 \cdot x_{1}+0 \cdot x_{2}+x_{3}=3\end{array}\right.$
   или   $\left\{\begin{array}{l}
x_{1}=-1 \\
x_{2}=1 \\
x_{3}=3
\end{array}\right.$

Ответ. $\left\{\begin{array}{l}
x_{1}=-1 \\
x_{2}=1 \\
x_{3}=3
\end{array}\right.$

Решение СЛАУ 4-ого порядка методом Гаусса, пример № 4

СЛАУ 3-его порядка:
1 —
2 —
3 —
4 —
5 —
6 —
7 —
8 —
9 —
10 —
11 —
12
СЛАУ 4-ого порядка:
1 —
2 —
3 —
4 —
5 —
6 —
7 —
8 —
9 —
10 —
11 —
12


Условие

 3x
1
 + x
2
 — 2x
3
 — 2x
4
  =  
-2
 2x
1
 — x
2
 + 2x
3
 + 2x
4
  =  
2
 2x
1
 + x
2
 — x
3
 — x
4
  =  
-1
 x
1
 + x
2
 — 3x
3
 + 2x
4
  =  
-3

Решение системы линейных алгебраических уравнений методом Гаусс

Для проверки ответов можете воспользоваться нашим онлайн сервисом —
Решение системы линейных уравнений методом Гаусса.
Все действия описанные в данном разделе не противоречат правилам обращения с матрицами и являются
элементарными преобразованиями матрицы.
Если после изучения
примеров решения задач
у Вас останутся вопросы, то Вы всегда можете задать их на
форуме, и не забывайте про наши
онлайн калькуляторы для
решения задач по математике и другим предметам!

Перепишем систему линейных алгебраических уравнений в матричную форму. Получится матрица 4 × 5,
слева от разделительной линии стоят коэффициенты при переменных, а справа стоят свободные члены.

Проведём следующие действия:

  • Поменяем местами строку № 1 и строку № 4

Получим:

Проведём следующие действия:

  • Из строки № 2 вычтем строку № 1 умноженную на 2 (Строка 2 — 2 × строка 1)
  • Из строки № 3 вычтем строку № 1 умноженную на 2 (Строка 3 — 2 × строка 1)
  • Из строки № 4 вычтем строку № 1 умноженную на 3 (Строка 4 — 3 × строка 1)

Получим:

Проведём следующие действия:

  • Строку № 3 умножим на -1 (Строка 3 = строка 3 * -1)
  • Поменяем местами строку № 2 и строку № 3

Получим:

Проведём следующие действия:

  • К строке № 3 прибавим строку № 2 умноженную на 3 (Строка 3 + 3 × строка 2)
  • К строке № 4 прибавим строку № 2 умноженную на 2 (Строка 4 + 2 × строка 2)

Получим:

Проведём следующие действия:

  • Строку № 4 поделим на -3 (Строка 4 = строка 4 / -3)
  • Поменяем местами строку № 3 и строку № 4

Получим:

Проведём следующие действия:

  • К строке № 4 прибавим строку № 3 умноженную на 7 (Строка 4 + 7 × строка 3)

Получим:

Проведём следующие действия:

  • Строку № 4 поделим на 55 (Строка 4 = строка 4 / 55)
  • Из строки № 3 вычтем строку № 4 умноженную на 6 (Строка 3 — 6 × строка 4)
  • Из строки № 2 вычтем строку № 4 умноженную на 5 (Строка 2 — 5 × строка 4)
  • Из строки № 1 вычтем строку № 4 умноженную на 2 (Строка 1 — 2 × строка 4)

Получим:

Проведём следующие действия:

  • К строке № 2 прибавим строку № 3 умноженную на 5 (Строка 2 + 5 × строка 3)
  • К строке № 1 прибавим строку № 3 умноженную на 3 (Строка 1 + 3 × строка 3)

Получим:

Проведём следующие действия:

  • Из строки № 1 вычтем строку № 2 (Строка 1 — строка 2)

Получим:

В левой части матрицы по главной диагонали остались одни единицы. В правом столбце получаем решение:

х1 = 0
х2 = 0
х3 = 1
х4 = 0

Вы поняли, как решать? Нет?

Помощь с решением

Метод Гаусса — примеры c решением, теоремы и формулы

Метод Гаусса – идеальный вариант для решения систем линейных алгебраических уравнений (далее СЛАУ). Благодаря методу Гаусса можно последовательно исключать неизвестные путём элементарных преобразований. Метод Гаусса – это классический метод решения СЛАУ, который и рассмотрен ниже.

Карл Фридрих Гаусс – немецкий математик, основатель одноименного метода решения СЛАУ

Карл Фридрих Гаусс – был известным великим математиком и его в своё время признали «королём математики». Хотя название «метод Гаусса» является общепринятым, Гаусс не является его автором: метод Гаусса был известен задолго до него. Первое его описание имеется в китайском трактате «Математика в девяти книгах», который составлен между II в. до н. э. и I в. н. э. и представляет собой компиляцию более ранних трудов, написанных примерно в X в. до н. э.

Метод Гаусса – последовательное исключение неизвестных. Этот метод используется для решения квадратных систем линейных алгебраических уравнений. Хотя уравнения при помощи метода Гаусса решаются легко, но всё же студенты часто не могут найти правильное решение, так как путаются в знаках (плюсы и минусы). Поэтому во время решения СЛАУ необходимо быть предельно внимательным и только тогда можно легко, быстро и правильно решить даже самое сложное уравнение.

У систем линейных алгебраических уравнений есть несколько преимуществ: уравнение не обязательно заранее на совместность; можно решать такие системы уравнений, в которых число уравнений не совпадает с количеством неизвестных переменных или определитель основной матрицы равняется нулю; есть возможность при помощи метода Гаусса приводить к результату при сравнительно небольшом количестве вычислительных операций.

Определения и обозначения

Как уже говорилось, метод Гаусса вызывает у студентов некоторые сложности. Однако, если выучить методику и алгоритм решения, сразу же приходит понимание в тонкостях решения.

Для начала систематизируем знания о системах линейных уравнений.

Обратите внимание!

СЛАУ в зависимости от её элементов может иметь:

  1. Одно решение;
  2. много решений;
  3. совсем не иметь решений.

В первых двух случаях СЛАУ называется совместимой, а в третьем случае – несовместима. Если система имеет одно решение, она называется определённой, а если решений больше одного, тогда система называется неопределённой.

Метод Крамера и матричный способ не подходят для решения уравнений, если система имеет бесконечное множество решений. Вот поэтому нам и нужен метод Гаусса, который поможет нам в любом случае найти правильное решение. К элементарным преобразованиям относятся:

  • перемена мест уравнений системы;
  • почленное умножение обеих частей на одно из уравнений на некоторое число, так, чтобы коэффициенты при первой переменной в двух уравнениях были противоположными числами;
  • сложение к обеим частям одного из уравнений определённых частей другого уравнения.

Итак, когда мы знаем основные правила и обозначения, можно приступать к решению.

Теперь рассмотрим, как решаются системы методом Гаусса на простом примере:

   

где а, в, с  – заданные коэффициенты, d – заданные свободные члены, x, y, z – неизвестные. Коэффициенты и свободные члены уравнения можно называть его элементами.

Если = = = , тогда система линейных алгебраических уравнений называется однородной, в другом случае – неоднородной.

Множественные числа , , называются решением СЛАУ, если при подстановке , , в СЛАУ получим числовые тождества.

Система, которую мы написали выше имеет координатную форму. Если её переделать в матричную форму, тогда система будет выглядеть так:

– это основная матрица СЛАУ.

– матрица столбец неизвестных переменных.

– матрица столбец свободных членов.

Если к основной матрице добавить в качестве – ого столбца матрицу-столбец свободных членов, тогда получится расширенная матрица систем линейных уравнений. Как правило, расширенная матрица обозначается буквой , а столбец свободных членов желательно отделить вертикальной линией от остальных столбцов. То есть, расширенная матрица выглядит так:

Если квадратная матрица равна нулю, она называется вырожденная, а если – матрица невырожденная.

Обратите внимание!

Если с системой уравнений:          

Произвести такие действия:

  • умножать обе части любого из уравнений на произвольное и отличное от нуля число ;
  • менять местами уравнения;
  • к обеим частям любого из уравнений прибавить определённые части другого уравнения, которые умножаются на произвольное число ,

тогда получается эквивалентная система, у которой такое же решение или нет решений совсем.

Теперь можно перейти непосредственно к методу Гаусса.

Простейшие преобразования элементов матрицы

Мы рассмотрели основные определения и уже понимаем, чем нам поможет метод Гаусса в решении системы. Теперь давайте рассмотрим простую систему уравнений. Для этого возьмём самое обычное уравнение, где и используем решение методом Гаусса:

Из уравнения запишем расширенную матрицу:

Из данной матрицы видно, по какому принципу она записана. Вертикальную черту не обязательно ставить, но просто так удобнее решать систему.

Определение

Матрица системы – это матрица, которая составляется исключительно с коэффициентами при неизвестных. Что касается расширенной матрицы системы, так, это такая матрица, в которой кроме коэффициентов записаны ещё и свободные члены. Любую из этих матриц называют просто матрицей.

На матрице, которая написана выше рассмотрим, какие существуют элементарные преобразования:

1. В матрице строки можно переставлять местами. Например, в нашей матрице спокойно можно переставить первую и вторую строки:

.

2. Если в матрице имеются (или появились) пропорциональные строки (одинаковые), тогда необходимо оставить всего лишь одну строку, а остальные убрать (удалить).

3. Если в ходе преобразований в матрице появилась строка, где находятся одни нули, тогда такую строку тоже нужно удалять.

4. Строку матрицы можно умножать (делить) на любое число, которое отличное от нуля. Такое действие желательно проделывать, так как в будущем проще преобразовывать матрицу.

5. Сейчас рассмотрим преобразование, которое больше всего вызывает затруднение у студентов. Для этого возьмём изначальную нашу матрицу:

Для удобства умножаем первую строку на (-3):

Теперь ко второй строке прибавляем первую строку, которую умножали на -3. Вот что у нас получается:

В итоге получилось такое преобразование:

Теперь для проверки можно разделить все коэффициенты первой строки на те же и вот что получается:

В матрице верхняя строка преобразовалась:

Первую строку делим на и преобразовалась нижняя строка:

И верхнюю строку поделили на то же самое число :

Как вы можете убедиться, в итоге строка, которую мы прибавляли ни капельки не изменилась, а вот вторая строка поменялась. ВСЕГДА меняется только та строка, к которой прибавляются коэффициенты.

Мы расписали в таких подробностях, чтобы было вам понятно, откуда какая цифра взялась. На практике, например, на контрольной или экзамене матрица так подробно не расписывается. Как правило, в задании решение матрицы оформляется так:

.

Обратите внимание!

Если в примере приведены десятичные дроби, метод Гаусса в этом случае также поможет решить систему линейных алгебраических уравнений. Однако, не стоит забывать, что следует избегать приближённых вычислений, так как ответ будет неверным. Лучше всего использовать десятичные дроби, а от них переходить к обыкновенным дробям.

Алгоритм решения методом Гаусса пошагово

После того, как мы рассмотрели простейшие преобразования, в которых на помощь пришёл метод Гаусса, можем вернуться к нашей системе, которую уже разложили по полочкам и пошагово распишем:

Шаг 1. Переписываем систему в виде матрицы

Записываем матрицу:

Шаг 2.

Преобразовываем матрицу: вторую строку в первом столбце приводим к нулю

Как мы привели вторую строку в первом столбце к нулю описано выше. Напомним, что первую строку умножали на и вторую строку прибавили к первой , умноженной на .

Шаг 3. Приводим матрицу к ступенчатому виду

Теперь вторую строку можно поделить на 2 и получается:

Верхнюю строку делим на и приводим матрицу к ступенчатому виду:

Когда оформляют задание, так и отчёркивают простым карандашом для упрощения работы, а также обводят те числа, которые стоят на “ступеньках”. Хотя в учебниках и другой литературе нет такого понятия, как ступенчатый вид. Как правило, математики такой вид называют трапециевидным или треугольным.

Шаг 4. Записываем эквивалентную систему

После наших элементарных преобразований получилась эквивалентная система:

Шаг 5. Производим проверку (решение системы обратным путём)

Теперь систему нужно решить в обратном направлении, то есть обратным ходом, начиная с последней строки. :

находим : ,

,

.

После находим :

,

.

Тогда:

.

Как видим, уравнение решено правильно, так как ответы в системе совпадают.

Решение систем линейных уравнений методом Гаусса, в которых основная матрица невырожденная, а количество в ней неизвестных равняется количеству уравнений

Как мы уже упоминали, невырожденная матрица бывает тогда, когда . Разберём систему уравнений невырожденной матрицы, где уравнений по количеству столько же, сколько и неизвестных. Эту систему уравнений решим другим способом.

Дана система уравнений:

Для начала нужно решить первое уравнение системы относительно неизвестной переменной . Далее подставим полученное выражение сначала во второе уравнение, а затем в третье, чтобы исключить из них эту переменную.

Теперь переходим ко второму уравнению системы относительно и полученный результат подставим в третье уравнение.. Это нужно для того, чтобы исключить неизвестную переменную :

Из последнего, третьего уравнения мы видим, что . Из второго уравнения находим . И последнее, находим первое уравнение .

Итак, мы нашли все три неизвестных при помощи последовательного исключения. Такой процесс называют – прямой ход метода Гаусса. Когда последовательно находятся неизвестные переменные, начиная с последнего уравнения, называется обратным ходом метода Гаусса.

Когда выражается через и в первом уравнении, а затем подставляется полученное выражение во второе или третье уравнения, тогда, чтобы привести в к такому же результату, необходимо проделать такие действия:

  • берём второе уравнение и к его левой и правой частям прибавляем определённые части из первого уравнения, которые умножаются на ,
  • берём третье уравнение и к его левой и правой частям прибавляем определённые части из первого уравнения, которые умножаются на .

И действительно, благодаря такой процедуре у нас есть возможность исключать неизвестную переменную со второго и третьего уравнения системы:

Возникают нюансы с исключением неизвестных переменных тогда, когда в уравнении системы нет каких-либо неизвестных переменных. Рассмотрим такую систему:

В этой системе в первом уравнении нет переменной и поэтому у нас нет возможности решить первое уравнение системы относительно , чтобы исключить данную переменную из остальных уравнений. В таком случае выход есть. Нужно всего лишь уравнения переставить местами.

Так как мы описываем уравнения системы, в которых определитель основных матриц отличен от нуля, тогда всегда есть такое уравнение, в котором есть необходимая нам переменная и это уравнение мы можем поставить туда, куда нам нужно.

В примере, который мы рассматриваем, достаточно всего лишь поменять местами первое и второе уравнение.

Теперь мы можем спокойно разрешить первое уравнение относительно переменной и убрать (исключить) из остальных уравнений в системе. Вот и весь принцип работы с такими, на первый взгляд, сложными системами.

Решение систем линейных уравнений методом Гаусса, в которых основная матрица вырожденная, а количество в ней неизвестных не совпадает с количеством уравнений

Метод Гаусса помогает решать системы уравнений, у которых основная матрица прямоугольная или квадратная, но основная вырожденная матрица может совсем не иметь решений, иметь бесконечное множество решений или иметь всего лишь одно единственное решение.

Рассмотрим, как при помощи метода Гаусса устанавливается совместность или несовместность систем линейных уравнений. В случае, если есть совместность определим все решения или одно решение.

В принципе, исключать неизвестные переменные можно точно так, как описано выше. Однако, есть некоторые непонятные ситуации, которые могут возникнуть в ходе решения:

1. На некоторых этапах в момент исключения неизвестных переменных некоторые уравнения могут обратиться в тождества . В данном случае такие уравнения лишние в системе и их можно смело полностью убирать, а затем продолжать решать уравнение методом Гаусса.

Например, вам попалась подобная система:

У нас получается такая ситуация

Как видим, второе уравнение . Соответственно, данное уравнение мы можем из системы удалить, так как оно без надобности.

Дальше можно продолжать решение системы линейных алгебраических уравнений уравнений традиционным методом Гаусса.

2. При решении уравнений прямым ходом методом Гаусса могут принять не только одно, но и несколько уравнений такой вид: , где – число, которое отличное от нуля. Это говорит о том, что такое уравнение никогда не сможет превратиться в тождество даже при любых значениях неизвестных переменных. То есть, можно выразить по-другому. Если уравнение приняло  вид, значит система несовместна, то есть, не имеет решений. Рассмотрим на примере:

Для начала необходимо исключить неизвестную переменную из всех уравнений данной системы, начиная со второго уравнения. Для этого нужно прибавить к левой и правой частям второго, третьего, четвёртого уравнения части (левую и правую) первого уравнения, которые соответственно, умножаются на (-1), (-2), (-3). Получается:

В третьем уравнении получилось равенство . Оно не подходит ни для каких значений неизвестных переменных , и , и поэтому, у данной системы нет решений. То есть, говорится, что система не имеет решений.

3. Допустим, что при выполнении прямого хода методом Гаусса нам нужно исключить неизвестную переменную , и ранее, на каком-то этапе у нас уже исключалась вместе с переменной . Как вы поступите в таком случае? При таком положении нам нужно перейти к исключению переменной . Если же  уже исключались, тогда переходим к ,  и т. д.

Рассмотрим систему уравнений на таком этапе, когда уже исключилась переменная :

Такая система уравнений после преобразования выглядит так:

Вы наверное уже обратили внимание, что вместе с исключились и . Поэтому решение методом Гаусса продолжаем исключением переменной из всех уравнений системы, а начнём мы с третьего уравнения:

Чтобы завершить уравнение прямым ходом метода Гаусса, необходимо исключить последнюю неизвестную переменную из последнего уравнения:

Допусти, что система уравнений стала:

В этой системе нет ни одного уравнения, которое бы сводилось к . В данном случае можно было бы говорить о несовместности системы. Дальше непонятно, что же делать? Выход есть всегда. Для начала нужно выписать все неизвестные, которые стоят на первом месте в системе:

В нашем примере это , и . В левой части системы оставим только неизвестные, которые выделены зелёным квадратом а в правую перенесём известные числа, но с противоположным знаком. Посмотрите на примере, как это выглядит:

Можно придать неизвестным переменным с правой части уравнений свободные (произвольные) значения: , , , где , ,  – произвольные числа.

Теперь в правых частях уравнений нашей системы имеются числа и можно приступать к обратному ходу решения методом Гаусса.

В последнем уравнении системы получилось: , и теперь мы легко найдём решение в предпоследнем уравнении: , а из первого уравнения получаем:

= =

В итоге, получился результат, который можно и записать.

Ответ

,

,

,

,

,

.

Примеры решения методом Гаусса

Выше мы подробно расписали решение системы методом Гаусса. Чтобы закрепить материал, решим несколько примеров, в которых опять нам поможет метод Гаусса. Соответственно, начнём с самой простой системы.

Пример 1

Задача 

Решить систему линейных алгебраических уравнений методом Гаусса:

Решение

Выписываем матрицу, куда добавляем столбец свободных членов:

Прежде всего мы смотрим на элемент, который находится в матрице в левом верхнем углу (первая строка, первый столбец). Для наглядности выделим цифру зелёным квадратом. На этом месте практически всегда стоит единица:

Так как мы должны использовать подходящее элементарное преобразование строк и сделать так, чтобы элемент, который находится в матрице под выделенной цифрой превратился в . Для этого можно ко второй строке прибавить первую строку и умножить на .Однако, не сильно хочется работать с дробями, поэтому давайте постараемся этого избежать. Для этого нужно вторую строку умножить на (разрешающий элемент данного шага).

Соответственно, первая строка остаётся неизменной, а вторая поменяется:

Подбираем такое элементарное преобразование строк, чтобы во второй строке в первом столбце образовался . Для этого первую строку нужно умножить на и только после этого ко второй строке прибавить изменённую после умножения на вторую строку. Вот что получилось:

. Теперь прибавляем со второй строки первую строку . У нас получился , который записываем во вторую строку в первый столбец. Также решаем и остальные элементы матрицы. Вот что у нас получилось:

Как всегда у нас первая строка осталась без изменений, а вторая с новыми числами.

Итак, у нас получился ступенчатый вид матрицы:

Записываем новую систему уравнений:

Для проверки решаем систему обратным ходом. Для этого находим сначала :

Так как найден, находим :

.

Подставляем в изначальную нашу систему уравнений найденные и :

и .

Как видите из решения, система уравнений решена верно. Запишем ответ.

Ответ

Выше мы решали систему уравнений в двумя неизвестными, а теперь рассмотрим систему уравнений с тремя неизвестными.

Пример 2

Задача

Решить систему уравнений методом Гаусса:

Решение

Составляем матрицу, куда вписываем и свободные члены:

Что нам надо? Чтобы вместо цифры 2 появился 0. Для этого подбираем ближайшее число. Например, можно взять цифру -2 и на неё перемножить все элементы первой строки. Значит, умножаем , а потом прибавляем, при этом задействуем вторую строку: . В итоге у нас получился нуль, который записываем во вторую строку в первый столбец. Затем , и . Аналогично, и . И умножаем свободный член . Так и запишем следующую матрицу. Не забывайте, что первая строка остаётся без изменений:

Дальше необходимо проделать те же самые действия по отношению к третьей строке. То есть, первую строку нужно умножать не на (-2), а на цифру 3, так как и в третьей строке нужно коэффициенты привести у нулю. Также первую строку умножаем на 3 и прибавляем третью строку. Получается так:

Теперь нужно обнулить элемент 7, который стоит в третьей строке во втором столбце. Для этого выбираем цифру (-7) и проделываем те же действия. Однако, необходимо задействовать вторую строку. То есть, вторую строку умножаем на (-7) и прибавляем с третьей строкой. Итак, . Записываем результат в третью строку. Такие же действия проделываем и с остальными элементами. Получается новая матрица:

В результате получилась ступенчатая система уравнений:

Сначала находим : ,

.

Обратный ход:

Итак, уравнение системы решено верно.

Ответ

,

,

.

Пример 3

Система с четырьмя неизвестными более сложная, так как в ней легко запутаться. Попробуем решить такую систему уравнений.

Задача

Решите систему уравнений методом Гаусса:

Решение                                                                

В уравнении , то есть – ведущий член и пусть  ≠ 0

Из данного уравнения составим расширенную матрицу:

Теперь нужно умножить последние три строки (вторую, третью и четвёртую) на: , , . Затем прибавим полученный результат ко второй, третьей и четвёртой строкам исключаем переменную из каждой строки, начиная не с первой, а не со второй. Посмотрите, как изменилась наша новая матрица и в теперь стоит 0.

Поменяем вторую и третью строку местами и получим:

Получилось так, что = b и тогда, умножая вторую строку на (-7/4) и результат данной строки, прибавляя к четвёртой, можно исключить переменную из третьей и четвёртой строк:

Получилась такая матрица:

Также, учитывая, что  = , умножим третью строку на: 13,5/8 = 27/16, и, полученный результат прибавим к четвёртой, чтобы исключить переменную и получаем новую систему уравнений:

Теперь необходимо решить уравнение обратным ходом и найдём из последнего, четвёртого уравнения ,

из третьего: = = =

второе уравнение находим: = = = 2,

из первого уравнения: = .

Значит, решение системы такое: (1, 2, -1, -2).

Ответ

,

,

,

.

Добавим ещё несколько примеров для закрепления материла, но без такого подробного описания, как предыдущие системы уравнений.

Пример 4

Задача

Решить систему уравнений методом Гаусса:

Решение

Записываем расширенную матрицу системы:

Сначала смотрим на левое верхнее число:

Как выше уже было сказано, на этом месте должна стоять единица, но не обязательно. Производим такие действия: первую строку умножаем на -3, а потом ко второй строке прибавляем первую:

 

Производим следующие действия: первую строку умножаем на -1. Затем к третьей строки прибавляем вторую:

Теперь вторую строку умножаем на 1, а затем к третьей строке прибавляем вторую:

Получился ступенчатый вид уравнения:

Проверяем:

,

,

,

,

.

.

  Ответ

,

,

.

Заключение

Итак, вы видите, что метод Гаусса – интересный и простой способ решения систем линейных алгебраических уравнений. Путём элементарных преобразований нужно из системы исключать неизвестные переменные, чтобы систему превратить в ступенчатый вид. Данный метод удобен тем, что всегда можно проверить, правильно ли решено уравнение. Нужно просто подставить найденные неизвестные в изначальную систему уравнений.

Если элементы определителя не равняются нулю, тогда лучше обратиться к методу Крамера, а если же элементы нулевые, тогда такие системы очень удобно решать благодаря методу Гаусса.

Предлагаем ещё почитать учебники, в которых также описаны решения систем методом Гаусса.

Литература для общего развития:

Умнов А. Е. Аналитическая геометрия и линейная алгебра, изд. 3: учеб. пособие – М. МФТИ – 2011 – 259 с.

Карчевский Е. М. Лекции по линейной алгебре и аналитической геометрии, учеб. пособие – Казанский университет – 2012 – 302 с.

Метод Гаусса – теорема, примеры решений обновлено: 16 апреля, 2020 автором: Научные Статьи.Ру

Метода Гаусса: примеры решения СЛАУ: понятия, определения, примеры задач

Найти решение этого же примера методом Гаусса в матричной форме записи:

3×1+2×2+x3+x4=-2×1-x2+4×3-x4=-1-2×1-2×2-3×3+x4=9×1+5×2-x3+2×4=4

Как решать?

Расширенная матрица системы представлена в виде:

   x1    x2     x3 x432111-14-1-2-2-3115-12-2-194

Прямой ход метода Гаусса в данном случае предполагает приведение расширенной матрицы к трапецеидальному виду при помощи элементарных преобразований. Этот процесс очень поход на процесс исключения неизвестных переменных в координатном виде.

Преобразование матрицы начинается с превращения всех элементов нулевые. Для этого к элементам 2-ой, 3-ей и 4-ой строк прибавляем соответствующие элементы 1-ой строки, которые умножены на -a21a11=-13, -a31a11=—23=23 и на -а41а11=-13.

Дальнейшие преобразования происходит по такой схеме: все элементы во 2-ом столбце, начиная с 3-ей строки, становятся нулевыми. Такой процесс соответствует процессу исключения переменной  . Для того, чтобы выполнить этой действие, необходимо к элементам 3-ей и 4-ой строк прибавить соответствующие элементы 1-ой строки матрицы, которая умножена на -а32(1)а22(1)=-23-53=-25 и -а42(1)а22(1)=-133-53=135:

   x1    x2     x3 x43211|-20-53113-43|-130-23-7353|2330133-4353|143~

      x1                 x2                           x3                           x4~3211|-20-53113-43|-130-23+(-25)(-53)-73+(-25)11353+(-25)(-43)|233+(-25)(-13)0133+135(-53)-43+135×11353+135(-43)|143+135(-13)~

       x1    x2     x3       x4~3211|-20-53113-43|-1300-195115|39500415-95|195

Теперь исключаем переменную x3 из последнего уравнения — прибавляем к элементам последней строки матрицы соответствующие элементы последней строки, которая умножена на а43(2)а33(2)=-415-195=4119.

       x1    x2     x3       x43211|-20-53113-43|-1300-195115|39500415-95|195~

      x1    x2               x3                           x4~3211|-20-53113-43|-1300-195115|39500415+4119(-195)-95+4119×115|195+4119×395~

       x1    x2     x3       x4~3211|-20-53113-43|-1300-195115|3950005619|39219

Теперь применим обратных ход метода. В матричной форме записи такое преобразование матрицы, чтобы матрица, которая отмечена цветом на изображении:

   x1    x2     x3       x43211|-20-53113-43|-1300-195115|3950005619|39219

стала диагональной, т.е. приняла следующий вид:

   x1    x2     x3       x43000|а10-5300|а200-1950|а30005619|39219, где а1, а2, а3 — некоторые числа.

Такие преобразования выступают аналогом прямому ходу, только преобразования выполняются не от 1-ой строки уравнения, а от последней. Прибавляем к элементам 3-ей, 2-ой и 1-ой строк соответствующие элементы последней строки, которая умножена на

-1155619=-209280, на —435619=1942 и на -15619=1956.

   x1    x2     x3       x43211|-20-53113-43|-1300-195115|3950005619|39219~

      x1    x2      x3                   x4~3211+(-1956)5619|-2+(-1956)392190-53113-43+1942×5619|-13+1942×3921900-195115+(-209280)5619|395+(-209280)392190005619|39219~

       x1    x2     x3       x4~3210|-90-531130|900-1950|-3850005619|39219

Далее прибавляем к элементам 2-ой и 1-ой строк соответствующие элементы 3-ей строки, которые умножены на

-113-195=5557 и на -1-195=519.

 x1    x2     x3       x43210|-90-531130|900-1950|-3850005619|39219~

      x1    x2             x3                   x4~321+519(-195)0|-9+519(-385)0-53113+5557(-195)0|9+5557(-385)00-1950|-3850005619|39219~

       x1    x2     x3       x4~3210|-110-5300|5300-1950|-3850005619|39219

На последнем этапе прибавляем элементы 2-ой строки к соответствующим элементам 1-ой строки, которые умножены на -2-53=65.

 x1    x2     x3       x43210|-110-5300|5300-1950|-3850005619|39219~

      x1           x2            x3      x4~32+65(-53)00|-11+65×53)0-5300|5300-1950|-3850005619|39219~

       x1    x2     x3       x4~3000|-90-5300|5300-1950|-3850005619|39219

Полученная матрица соответствует системе уравнений

3×1=-9-53×2=53-195×3=-3855619×4=39219, откуда находим неизвестные переменные.

Ответ: x1=-3, x2=-1,x3=2,x4=7.​​​

Метод Гаусса. Примеры

Метод Гаусса заключается в последовательном исключении переменных и преобразовании системы линейных алгебраических уравнений

к треугольному виду

Предположим, что в системе коэффициент . Если это условие не выполняется, то на первое место переносим уравнение, которое ее удовлетворяет. С помощью первого уравнения исключим из остальных уравнений.

Для этого делят первую строчку на , обозначим

.

Дальше второй строки вычитаем первую строку, умноженную на ;от третьего первую строчку, умноженный на ; и так далее до последней строки. Получим таблицу коэффициентов:

Для неизвестных имеем систему уравнений. Выполняя, как и раньше, исключим из всех уравнений, начиная с третьего. Для этого сначала разделим вторую строчку на .

Если коэффициент , то переставим уравнения так, чтобы выполнялось условие .

Обозначив

,

от третьей строки вычтем вторую строчку, умноженный на ;

от четвертой строки вычтем вторую строчку, умноженный на и т.д. Получим таблицу коэффициентов:

Продолжая процесс исключения неизвестных получим таблицу:

Таблица коэффициентов при неизвестных сводится к треугольному виду. Все главной диагонали элементы . Запишем соответствующую систему уравнений:

Переход от первой системы уравнений до последней называется прямым ходом метода Гаусса. Обратный ход метода Гаусса начинается с последней системы уравнений. Ее решают с конца до начала. Из последнего уравнения находят . Подставив это значение в предпоследнее — находят и т.д. Из первого уравнения находят .

Если система уравнений с неизвестными имеет единственное решение, то эта система всегда может быть преобразована к треугольному виду. Для студентов не всегда требуют, чтобы диагональные элементы были равны единице. Достаточно просто свести систему линейных уравнений к верхней треугольной.

———————————————

Пример 1.

Дана система трех линейных уравнений с тремя неизвестными. Решить систему методом Гаусса.

Решение.

Исключим неизвестную из второго и третьего уравнения. Для этого от них вычтем первое умноженное на

Видим, что наше уравнение в таком виде можно решать обратным ходом метода Гаусса. Для этого из последнего уравнения выразим

Подставим полученное значение в предыдущее уравнение и найдем

Из первого уравнения находим

Решение данной системы равен

——————————————

В случаях систем больших размеров, а также для удобства, часто на практике используют другую схему решения. Вместо преобразований над системой выполняют соответствующие преобразования над матрицей, составленной из коэффициентов при неизвестных и столбца из свободных членов, который для удобства выделяют вертикальной линией. Такую матрицу называют расширенной матрицей системы.

——————————————

Пример 2.

Решить систему четырех линейных алгебраических уравнений методом Гаусса.

Решение.

Выпишем расширенную матрицу для данной системы

Сведем ее к треугольному виду с помощью элементарных преобразований.

1.Поменяем местами первый и второй строки.

2. Добавим к элементам второго, третьего и четвертого строк элементы первой строки, умноженные соответственно на

3. Поменяем местами второй и третий строки. Добавим к элементам третьего и четвертого строк элементы второй строки, умноженные соответственно на

4. От четвертого уравнения умноженного на вычитаем третье уравнение умноженное на

Такой расширенной матрицы соответствует следующая система уравнений

С четвертого уравнения находим и подставляем в третье уравнение

Найденные значения подставляем во второе уравнение

Из первого уравнения находим первую неизвестную

Система полностью решена и – ее решение.

——————————————————

Посмотреть материалы:

Решение систем линейных уравнений методом Гаусса

Содержание:

Решение систем линейных уравнений методом Гаусса

  • Решение системы линейных уравнений Метод Гаусса (1) Одним из наиболее универсальных и эффективных методов решения систем линейной алгебры является метод Гаусса. i + ai2X2 H —— h alnxn = bi a22 x2 t ‘• * T a2n xn- » am2®2 + •• + ​​GmUn = bn • Где (t, j = 2, m) — новое значение коэффициента, Правильная часть получена после первого шага. Аналогичным образом исключают неизвестные X2 из всех уравнений системы, учитывая основной элемент <4UФ0, исключая первое и второе.

    Примеры решения и задачи с методическими указаниями

    Решение задачЛекции
    Сборник и задачникУчебник
    • Продолжайте этот процесс как можно больше. Если процесс приведения системы (1) к постепенной форме показывает нулевые уравнения, то есть уравнения вида 0 = 0, они отбрасываются. Если отображается уравнение вида 0 = aΦ0>, это указывает на несовместимость системы. Второй шаг (обратный) — это решение ступенчатой ​​системы. В общем, существует множество решений системы градуированных уравнений. В последнем уравнении этой системы первое неизвестное xb представлено оставшимися неизвестными (£ fc + 1, …, xn). Затем подставьте значение Xk в предпоследнее уравнение системы и выразите Xk- \ через a: n).

    Тогда найди Xk-2> … Примечания: 1. Если система ступеней представляет собой треугольник, то есть k = 7i, исходная система имеет единственное решение. Найти xn из последнего уравнения и из второго уравнения xn-1) из последнего далее в систему всех остальных неизвестных [xn — 2? ••• yXi). 2.

    Прибавьте произвольные значения к свободным неизвестным …, xn), получите бесконечное число решений для системы.

    Людмила Фирмаль

    На практике удобнее выполнять все базовые преобразования для строк, используя матрицу расширения, а не систему (1). Удобно, если коэффициент aj равен 1 (переместить уравнение на место или отделить обе стороны уравнения все ф 1). Пример: 1) Решить систему, используя метод Гаусса. 2x \ -x-2 + 3×3-5 # 4 = 1, X \ -X2-bx3 = 2 3xi-2×2-2hz-5×4 = 3, 7xi-5×2-9hz-10×4 = 8.

    ♦ В результате базового преобразования в расширенную матрицу системы / 2-1 3 «-5 1 \ 1-1-5 0 2 3 -2 -2 -5 3 \ 7-5-9-10 8 / 1 -1 -5 0 2 \ 0 1 13 -5 -3 0 1 13 -5 -3 х0 2 26-10-6 / ^ 1 -1 -5 0 2 \ 2-13 -5 1 3 -2 -2 -5 3 ^ 7-5-9-10 8J -1 О 1 Ах ах \ 0 O -5 0 ‘2 л 13-5-3 LLC O O O y Оригинальная система была уменьшена до ступенчатой системы. xi-x2-5xs = 2 x2 + 13 Гц + 5×4 = -3. Итак, общее решение системы: x2 = -5×4-13x-X \ = -5×4-8×3-1. 1, x2 = x3 = 0, x4 = 0. 2) Решить систему, используя метод Гаусса. — = О, -3, ♦ X1 + x2 + x3 = 3, 2xi + 3×2 + 3×3 = 7, 3X] + X2 + x3 = 5, 5xi-x2-. Xs = 3. ♦ Выполнять базовые преобразования в строках расширенной матрицы системы.

    / 11 1 3 \ / 11 1 3 \ / 1 1 1 3 \ / 1 1 1 3 \ 2337 010 1 0101 0101 31 15 ~ 0-2—2-4 ~ 0112 ~ 0011 \ 5 -1 -1 3 / \ 0 -b -6 -12 / \ 0 I 1 2 / \ 0 0 0 0 / Полученная матрица соответствует системе + X-2 + xs = 3, X-2 = 1 Xb = 1. Выполнение обратного хода приводит к £ 3 = 1, x2-1, Xj = 1.

    2.3.6. Примеры решения задач по теме «Системы уравнений общего вида. Мет

    Задача 1.

    Указать базисный минор матрицы

    Указание

    Определите вначале ранг матрицы А, а затем найдите ненулевой минор, порядок которого равен R(A).

    Решение

    Определим R(A). Вторая и четвертая строки А равны, поэтому после вычитания из 4-й строки 2-й получаем:

    Вычислим минор полученной матрицы, составленный из первых трех столбцов:

    Таким образом, найден минор максимально возможного (3-го) порядка, не равный нулю. Следовательно, ранг матрицы А равен рангу преобразованной матрицы, то есть равен 3, а рассмотренный минор является базисным.

    Ответ:

    Задача 2.

    Определить количество решений системы линейных уравнений

    .

    Указание

    Сравните ранги матрицы системы и расширенной матрицы.

    Решение

    Сравним ранги матрицы системы

    И расширенной матрицы

    .

    Для удобства вычислений будем искать ранг матрицы А1, отделив ее последний столбец вертикальной чертой. Тогда столбцы, стоящие слева от черты, образуют матрицу А, и мы одновременно найдем ранги обеих матриц.

    А1 ~ .

    Вычтем из второй строки удвоенную первую, а из третьей – первую, умноженную на 3:

    А1 ~ ~ .

    Таким образом, R(A) = 2, a R(A1) = 3, следовательно, система не имеет решений.

    Ответ: система несовместна.

    Задача 3.

    Найти общее решение линейной системы

    .

    Указание

    Убедившись в том, что система совместна, определите базисные и свободные неизвестные и выразите базисные неизвестные через свободные.

    Решение

    Найдем R(A) и R(A1):

    Итак, R = R(A) = R(A1) = 2, а число неизвестных П = 5. Следовательно, R < N, и система имеет бесконечно много решений (совместна, но не определена).

    Число базисных неизвестных равно R, то есть двум. Выберем в качестве базисных неизвестных Х1 и Х2, коэффициенты при которых входят в базисный минор преобразованной матрицы А: .

    Соответственно Х3, Х4, Х5 – свободные неизвестные.

    Запишем систему, равносильную исходной, коэффициентами в которой являются элементы полученной матрицы:

    И выразим базисные неизвестные через свободные:

    .

    Получено общее решение системы. Одно из частных решений можно найти, положив все свободные неизвестные равными нулю: Х3 = Х4 = Х5 = 0. Тогда

    Ответ:

    Задача 4.

    Найти общее решение системы, выразив в ответе первые неизвестные через последние:

    Указание

    Приведите расширенную матрицу к виду

    Решение

    Минор, состоящий из первых трех столбцов полученной матрицы,

    Поэтому R(A) = R(A1) = 3, выбранный минор является базисным, а Х1, Х2, Х3, коэффициенты при которых составляют базисный минор, – базисными неизвестными. Тогда свободное неизвестное – Х4, и система, равносильная исходной, имеет вид:

    Откуда

    Ответ:

    Задача 5.

    Найти фундаментальную систему решений однородной линейной системы

    Указание

    Количество решений, образующих фундаментальную систему, равно числу

    Свободных неизвестных. Задайте свободным неизвестным значения 1,0,0; 0,1,0; 0,0,1 и вычислите соответствующие значения базисных неизвестных.

    Решение

    Количество решений, образующих фундаментальную систему, равно числу

    Свободных неизвестных.

    Матрица А1 отличается от матрицы А только добавлением нулевого столбца свободных членов, поэтому все ее ненулевые миноры являются минорами матрицы А, то есть R(A) = R(A1). Найдем R(A):

    Выберем в качестве базисного минора

    Значит, R(A) = 2. Пусть Х4, Х5 – базисные неизвестные, Х1, Х2, Х3 – свободные неизвестные. Запишем для них новую систему:

    Откуда

    Фундаментальная система решений состоит из трех столбцов. Рассмотрим три набора значений свободных неизвестных:

    1) Х1 = 1, Х2 = Х3 = 0.

    Тогда Х4 = -0,2, Х5 = 1,2, и решение можно записать в виде столбца

    2) Х1 = 0, Х2 = 1, Х3 = 0.

    При этом Х4 = 1,2, Х5 = 3,8, и следующее решение системы имеет вид

    3) Х1 = Х2 = 0, Х3 = 1. Отсюда Х4 = -0,8, Х5 = -0,2, и последний столбец

    Фундаментальная система решений, построенная при таком выборе свободных неизвестных, называется Нормальной. Поскольку столбцы свободных неизвестных , , линейно независимы, это гарантирует линейную независимость решений Х1, Х2, Х3.

    Итак, в качестве фундаментальной системы решений можно выбрать

    При этом любое решение данной системы имеет вид: Х = с1Х1 + С2Х2 + С3Х3, где С1, С2, С3 – произвольные постоянные. Эта формула задает общее решение системы.

    Ответ:

    Задача 6.

    Составить однородную систему из двух уравнений, для которой столбцы

    Образуют фундаментальную систему решений.

    Указание

    Пусть искомая система имеет вид:

    Подставьте вместо Х1, …, Х5 элементы столбцов Х1, Х2, Х3 и решите полученную систему уравнений для коэффициентов Aij.

    Решение

    Существует бесконечно много систем однородных линейных уравнений, для каждой из которых фундаментальная система решений имеет указанный вид. Число уравнений в таких системах может быть различным. При этом можно указать их наименьшее требуемое количество, а увеличивать их число можно неограниченно.

    Определим вначале, из какого наименьшего числа уравнений может состоять такая система.

    Число элементов каждого столбца равно пяти, следовательно, в системе пять неизвестных (П = 5). Количество столбцов, составляющих фундаментальную систему, равно трем, то есть N R = 3, поэтому R = 5 – 3 = 2. Значит, матрица А должна иметь по крайней мере 2 строки. Следовательно, система уравнений с заданной фундаментальной системой решений может состоять из двух и более уравнений.

    Пусть искомая система имеет вид:

    Подставим вместо Х1, …, Х5 элементы столбцов Х1, Х2, Х3. Получим:

    Разобьем полученные 6 уравнений на две системы, одна из которых содержит A1I, а вторая – A2I:

    Найдем какое-либо частное решение этой системы. Приведем ее матрицу к треугольному виду:

    Откуда

    Следовательно,

    Выберем А14 = А15 = 4, тогда А11 = 0, А12 = 8, А13 = -4.

    2) Так же выглядит общее решение системы для A2I:

    Выберем свободные неизвестные так, чтобы получить решение, линейно независимое с предыдущим.

    Пусть А24 = 4, А25 = 0, тогда А21 = 5, А22 = 5, А23 = -3.

    Итак, используя найденные значения коэффициентов, можно составить линейную однородную систему:

    Фундаментальная система решений которой имеет вид, приведенный в условии задачи.

    Ответ:

    Задача 7.

    Найти общее решение неоднородной линейной системы

    С помощью фундаментальной системы решений соответствующей однородной системы.

    Указание

    Убедитесь в том, что система совместна. Затем составьте соответствующую однородную систему и найдите для нее фундаментальную систему решений. Далее используйте то, что общее решение неоднородной системы линейных уравнений является суммой общего решения соответствующей однородной системы и частного решения неоднородной системы.

    Решение

    Убедимся в том, что система совместна:

    Итак, R(A) = R(A1) = 2 – система совместна.

    Составим по преобразованной матрице однородную систему:

    И найдем для нее фундаментальную систему решений:

    Фундаментальная система решений может быть выбрана так:

    Общее решение неоднородной системы линейных уравнений является суммой общего решения соответствующей однородной системы и частного решения неоднородной системы.

    Теперь найдем какое-нибудь частное решение неоднородной системы

    Положим Х3 = Х4 = Х5 = 0, тогда . Следовательно,

    и общее решение системы имеет вид:

    Х = с1Х1 + С2Х2 + С3Х3 + Хчастн, где С1, С2, С3 – произвольные постоянные.

    Ответ:

    Задача 8.

    Решить систему методом Гаусса:

    .

    Указание

    Поменяйте местами 1-е и 2-е уравнения, чтобы в первом уравнении коэффициент при Х равнялся единице, а затем исключите Х из второго и третьего уравнений.

    Решение

    Метод Гаусса заключается в последовательном исключении неизвестных из уравнений системы. Для удобства его применения поменяем местами 1-е и

    2-е уравнения, чтобы в первом уравнении коэффициент при Х равнялся единице:

    Теперь исключим Х из второго и третьего уравнений. Для этого вычтем из второго уравнения первое, умноженное на 3, а из третьего – первое, умноженное на 2:

    Далее можно легко исключить Z из третьего уравнения, если прибавить к нему второе:

    Из последнего уравнения получаем, что У = 0. Подставляя это значение в первое и второе уравнения, находим остальные неизвестные: Z = 3, Х = 1.

    Ответ: Х = 1, У = 0, Z = 3.

    При применении метода Гаусса совсем не обязательно приводить систему к «классическому» треугольному виду:

    .

    Достаточно, чтобы матрица коэффициентов, например, системы трех уравнений с тремя неизвестными содержала два нуля в одном столбце и одновременно два нуля в одной строке, причем один из нулей стоял на пересечении этих строки и столбца.

    Задача 9.

    Решить систему методом Гаусса:

    Указание

    Исключите Х2 из 2-го и 4-го уравнений, используя 1-е уравнение, а затем вычтите из 3-го уравнения 2-е, чтобы исключить Х3.

    Решение

    Исключим Х2 из 2-го и 4-го уравнений. Для этого из 2-го уравнения вычтем 1-е, а к 4-му прибавим 1-е, умноженное на 2:

    Вычтем из 3-го уравнения 2-е, чтобы исключить Х3:

    Теперь вычтем из 4-го уравнения удвоенное 3-е:

    Из последнего уравнения находим . Тогда из 3-го уравнения Х1 = 0, из 2-го , из 1-го Х2 = 2.

    Ответ:

    < Предыдущая   Следующая >

    Систем линейных уравнений: исключение Гаусса

    Системы
    линейных уравнений:
    Решение методом исключения Гаусса
    (стр.
    6 из 7)

    Разделы: Определения,
    Решение по графику, Подстановка,
    Исключение / добавление, исключение по Гауссу.


    Решение трех переменных,
    линейных систем с тремя уравнениями сложнее, по крайней мере, на начальном этапе, чем
    решение систем с двумя переменными, потому что требуемые вычисления
    более грязный.Вам нужно будет очень аккуратно работать, и вам следует
    планируйте использовать много бумаги для заметок. Метод решения этих систем
    является расширением метода сложения двух переменных, поэтому сделайте
    конечно ты знаешь это
    метод хорошо и
    можно использовать его последовательно правильно.

    Хотя метод решения
    основан на добавлении / исключении, попытка выполнить фактическое добавление имеет тенденцию
    становится очень запутанным, поэтому существует систематизированный метод решения трех или более переменных
    системы.Этот метод называется «исключением по Гауссу» (с
    уравнения заканчиваются так называемой «строковой формой»).

    Начнем с простого, и
    работаем над более сложными примерами.

    • Решите следующие проблемы
      система уравнений.
    • Достаточно легко увидеть
      как действовать в этом случае. Я просто подставлю обратно значение z -value
      из третьего уравнения во второе, решите результат для
      л , г.
      а затем подключите z
      и y в
      первое уравнение и решите результат для x .

        10 л
        3 (3) = 11
        10 y 9 = 11
        10 y = 20
        y
        = 2

        5x +
        4 (2) (3) = 0
        5 x + 8 3 = 0
        5 x + 5 = 0
        5 x = 5
        x = 1

      Тогда решение
      ( х ,
      y , z ) = (1, 2, 3).

    Причина, по которой эта система была
    Легко решить, что система была «треугольной»; это относится
    к уравнениям, имеющим форму треугольника, из-за нижних уравнений
    содержащий только более поздние переменные.

      Дело в том, что в этом
      формат, система проста в решении. А гауссовское исключение — это
      метод, который мы будем использовать для преобразования систем в эту верхнетреугольную форму, используя
      операции со строками, которые мы изучили, когда применили метод сложения.

      • Решите следующие проблемы
        система уравнений с использованием исключения Гаусса.
      • Уравнение не решается
        для переменной, поэтому мне нужно будет выполнить умножение и сложение
        чтобы упростить эту систему. Чтобы отслеживать свою работу, напишу
        вниз на каждом шагу, когда я иду. Но я буду делать свои вычисления на бумаге для заметок.
        Вот как я это сделал:

        Первое, что нужно сделать
        состоит в том, чтобы избавиться от ведущих x -термов
        в два ряда.А пока я просто посмотрю, какие строки будут легко
        расчистить; Я могу поменять строки позже, чтобы перевести систему в «верхний
        треугольной «формы. Нет правила, которое гласит, что я должен использовать
        x — срок
        из первой строки, и в этом случае, думаю, будет проще
        используйте термин x
        из третьей строки, так как его коэффициент просто «1».
        Так что я умножу третью строку на 3,
        и добавьте его в первую строку.Я делаю вычисления на бумаге для заметок:

          … а потом записываю
          результатов:

            (Когда мы решали
            системы с двумя переменными, мы могли умножить строку, переписав систему
            в сторону, а затем добавить. Для этого нет места в
            система с тремя переменными, поэтому нам и нужна бумага для заметок.) ​​

            Предупреждение: поскольку я не
            на самом деле ничего не делаю с третьей строкой, я скопировал ее без изменений,
            в новую матрицу уравнений.Я б / у
            третий ряд, но я на самом деле не менял
            Это. Не путайте «использование» с «изменением».

            Чтобы получить меньшие числа
            для коэффициентов умножу первую строку пополам:

              Теперь умножу
              третий ряд на 5
              и добавьте это ко второму
              строка. Работаю на бумаге для заметок:

                … а потом записываю
                результаты: Авторские права
                Элизабет Стапель 2003-2011 Все права защищены

                  Я ничего не делал
                  с первым рядом, поэтому я скопировал его без изменений. Я работал с
                  третий ряд, но я работал только на
                  вторая строка, поэтому вторая строка обновляется, а третья строка копируется
                  более без изменений.

                  Хорошо, теперь
                  x — столбец
                  удаляется, за исключением ведущего члена в третьей строке.Так что дальше
                  Приходится работать над колонкой и .

                  Предупреждение: С третьего
                  уравнение имеет член x ,
                  Я больше не могу использовать его ни в одном из двух других уравнений (или я
                  отменить мой прогресс). Я могу работать с уравнением, но не с
                  Это.

                  Если я добавлю в два раза больше первого
                  строки во вторую строку, это даст мне ведущую 1
                  во втором ряду.Я не буду
                  избавились от ведущего y -терм
                  во втором ряду, но я его преобразовал (не вмешиваясь
                  дробями) в более простую форму. (Вы должны сохранить
                  обратите внимание на такого рода упрощения.) Сначала я делаю царапину
                  работа:

                    … а потом записываю
                    результатов:

                    Теперь могу использовать второй
                    ряд, чтобы убрать и -семестр
                    в первом ряду.Вторую строку умножу на 7
                    и добавить. Сначала я царапаю
                    работа:

                      … а потом записываю
                      результатов:

                        Я могу сказать что
                        z
                        сейчас, но для большей точности я разделю первую строку на
                        43.
                        Затем я переставляю ряды, чтобы придать им верхнетреугольную форму:

                          Теперь я могу начать процесс
                          обратного решения:

                          Тогда решение
                          ( х ,
                          y , z ) = ( 2,
                          3, 1
                          ) .

                          Примечание: нет ничего священного
                          о шагах, которые я использовал при решении указанной выше системы; там ничего не было
                          особенно о том, как я решил эту систему. Вы могли бы работать в другом
                          упорядочивайте или упрощайте разные строки, и все равно получите правильный ответ.
                          Эти системы достаточно сложны, поэтому вряд ли
                          один правильный способ вычисления ответа. Так что не беспокойтесь о том, «как
                          она знала, что делать дальше? », потому что здесь нет правила.я просто
                          делал все, что пришло мне в голову; Я делал то, что казалось самым простым или что-то еще
                          пришла в голову первая. Не волнуйтесь, если бы вы использовали совершенно другой
                          шаги. Если каждый шаг на этом пути верен, вы придумаете
                          Такой же ответ.


                          В приведенном выше примере я мог
                          пошли дальше в своих вычислениях и более тщательно проработали
                          строковые операции, очищая все термины и
                          кроме этого во втором ряду и во всех терминах z
                          кроме того, что в первой строке.Это то, что процесс тогда
                          посмотрели так:

                            Так я могу просто читать
                            от значений x ,
                            л , г.
                            и z ,
                            и мне не нужно возиться с обратной заменой. Это более полное
                            метод решения называется «методом исключения Гаусса-Жордана» (с
                            уравнения, заканчивающиеся тем, что называется «эшелон сокращенного ряда»
                            форма»).Многие тексты доходят до исключения Гаусса, но я
                            всегда было легче продолжать и делать Гаусс-Джордан.

                            Обратите внимание, что я выполнил две строковые операции
                            сразу на этом последнем шаге перед переключением строк. Пока я не
                            работая с и работая на в той же строке на том же шаге,
                            это нормально. В этом случае я работал с первой строкой и
                            рабочая по второй и третий ряды.

                            << Предыдущая Вверх | 1
                            | 2 | 3 | 4
                            | 5 | 6 | 7
                            |
                            Вернуться к указателю Далее
                            >>

                            Цитируйте эту статью
                            как:

                            Стапель, Елизавета.
                            «Системы линейных уравнений, решаемые методом исключения Гаусса».
                            Purplemath
                            Доступно по телефону https: // www.purplemath.com/modules/systlin6.htm .
                            Доступ [Дата] [Месяц] 2016 г.

                            Решение системы с исключением Гаусса

                            Результаты обучения

                            • Используйте метод исключения Гаусса для решения системы уравнений, представленной в виде расширенной матрицы.
                            • Интерпретировать решение системы уравнений, представленной в виде расширенной матрицы.

                            Мы увидели, как написать систему уравнений с расширенной матрицей , а затем, как использовать строковые операции и обратную подстановку, чтобы получить форму эшелон строки .Теперь мы будем использовать метод исключения Гаусса как инструмент для решения системы, записанной в виде расширенной матрицы. В нашем первом примере мы покажем вам процесс использования исключения Гаусса в системе двух уравнений с двумя переменными.

                            Пример: решение системы 2 X 2 методом исключения Гаусса

                            Решите данную систему методом исключения Гаусса.

                            [латекс] \ begin {array} {l} 2x + 3y = 6 \ hfill \\ \ text {} x-y = \ frac {1} {2} \ hfill \ end {array} [/ latex]

                            Показать решение

                            Сначала мы запишем это как расширенную матрицу.

                            [латекс] \ left [\ begin {array} {rr} \ hfill 2 & \ hfill 3 \\ \ hfill 1 & \ hfill -1 \ end {array} \ text {} | \ text {} \ begin {array} { r} \ hfill 6 \\ \ hfill \ frac {1} {2} \ end {array} \ right] [/ latex]

                            Нам нужна 1 в строке 1, столбце 1. Этого можно добиться, поменяв местами строку 1 и строку 2.

                            [латекс] {R} _ {1} \ leftrightarrow {R} _ {2} \ to \ left [\ begin {array} {rrr} \ hfill 1 & \ hfill -1 & \ hfill \\ \ hfill 2 & \ hfill 3 & \ hfill \ end {array} | \ begin {array} {rr} \ hfill & \ hfill \ frac {1} {2} \\ \ hfill & \ hfill 6 \ end {array} \ right] [/ latex]

                            Теперь у нас есть 1 как первая запись в строке 1, столбце 1.Теперь давайте получим 0 в строке 2, столбце 1. Это можно сделать, умножив строку 1 на [latex] -2 [/ latex], а затем прибавив результат к строке 2.

                            [латекс] -2 {R} _ {1} + {R} _ {2} = {R} _ {2} \ to \ left [\ begin {array} {rrr} \ hfill 1 & \ hfill -1 & \ hfill \\ \ hfill 0 & \ hfill 5 & \ hfill \ end {array} | \ begin {array} {rr} \ hfill & \ hfill \ frac {1} {2} \\ \ hfill & \ hfill 5 \ end {массив } \ right] [/ latex]

                            У нас есть только один шаг, чтобы умножить строку 2 на [latex] \ frac {1} {5} [/ latex].

                            [латекс] \ frac {1} {5} {R} _ {2} = {R} _ {2} \ to \ left [\ begin {array} {rrr} \ hfill 1 & \ hfill -1 & \ hfill \ \ \ hfill 0 & \ hfill 1 & \ hfill \ end {array} | \ begin {array} {cc} & \ frac {1} {2} \\ & 1 \ end {array} \ right] [/ latex]

                            Использовать обратную замену.Вторая строка матрицы представляет [латекс] y = 1 [/ латекс]. Подставьте обратно [latex] y = 1 [/ latex] в первое уравнение.

                            [латекс] \ begin {array} {l} x- \ left (1 \ right) = \ frac {1} {2} \ hfill \\ \ text {} x = \ frac {3} {2} \ hfill \ end {array} [/ latex]

                            Решение — точка [латекс] \ left (\ frac {3} {2}, 1 \ right) [/ latex].

                            Попробуй

                            Решите данную систему методом исключения Гаусса.

                            [латекс] \ begin {массив} {l} 4x + 3y = 11 \ hfill \\ \ text {} \ text {} \ text {} x — 3y = -1 \ hfill \ end {array} [/ latex]

                            Показать решение

                            [латекс] \ влево (2,1 \ вправо) [/ латекс]

                            В нашем следующем примере мы решим систему двух уравнений с двумя зависимыми переменными.Напомним, что зависимая система имеет бесконечное количество решений, и результатом операций со строками в ее расширенной матрице будет уравнение, такое как [latex] 0 = 0 [/ latex]. Мы также рассмотрим написание общего решения для зависимой системы.

                            Пример: решение зависимой системы

                            Решите систему уравнений.

                            [латекс] \ begin {array} {l} 3x + 4y = 12 \\ 6x + 8y = 24 \ end {array} [/ latex]

                            Показать решение

                            Выполните строковые операции на расширенной матрице, чтобы попытаться получить строковую форму .

                            [латекс] A = \ left [\ begin {array} {llll} 3 \ hfill & \ hfill & 4 \ hfill & \ hfill \\ 6 \ hfill & \ hfill & 8 \ hfill & \ hfill \ end {array} | \ begin {array} {ll} \ hfill & 12 \ hfill \\ \ hfill & 24 \ hfill \ end {array} \ right] [/ latex]

                            [латекс] \ begin {array} {l} \ hfill \\ \ begin {array} {l} — \ frac {1} {2} {R} _ {2} + {R} _ {1} = { R} _ {1} \ to \ left [\ begin {array} {llll} 0 \ hfill & \ hfill & 0 \ hfill & \ hfill \\ 6 \ hfill & \ hfill & 8 \ hfill & \ hfill \ end { array} | \ begin {array} {ll} \ hfill & 0 \ hfill \\ \ hfill & 24 \ hfill \ end {array} \ right] \ hfill \\ {R} _ {1} \ leftrightarrow {R} _ {2} \ to \ left [\ begin {array} {llll} 6 \ hfill & \ hfill & 8 \ hfill & \ hfill \\ 0 \ hfill & \ hfill & 0 \ hfill & \ hfill \ end {array} | \ begin {array} {ll} \ hfill & 24 \ hfill \\ \ hfill & 0 \ hfill \ end {array} \ right] \ hfill \ end {array} \ hfill \ end {array} [/ latex]

                            Матрица заканчивается всеми нулями в последней строке: [latex] 0y = 0 [/ latex].Таким образом, существует бесконечное количество решений и система классифицируется как зависимая. Чтобы найти общее решение, вернитесь к одному из исходных уравнений и решите для [latex] y [/ latex].

                            [латекс] \ begin {array} {l} 3x + 4y = 12 \ hfill \\ \ text {} 4y = 12 — 3x \ hfill \\ \ text {} y = 3- \ frac {3} {4} x \ hfill \ end {array} [/ latex]

                            Итак, решение этой системы — [латекс] \ left (x, 3- \ frac {3} {4} x \ right) [/ latex].

                            Теперь мы перейдем на ступенчатую форму, чтобы решить систему линейных уравнений 3 на 3.Общая идея состоит в том, чтобы исключить все переменные, кроме одной, с помощью операций со строками, а затем выполнить обратную замену для поиска других переменных.

                            Пример: решение системы линейных уравнений с использованием матриц

                            Решите систему линейных уравнений с помощью матриц.

                            [латекс] \ begin {массив} {c} \ begin {array} {l} \ hfill \\ \ hfill \\ x-y + z = 8 \ hfill \ end {array} \\ 2x + 3y-z = -2 \\ 3x — 2y — 9z = 9 \ end {array} [/ latex]

                            Показать решение

                            Сначала мы пишем расширенную матрицу.

                            [латекс] \ left [\ begin {array} {rrr} \ hfill 1 & \ hfill -1 & \ hfill 1 \\ \ hfill 2 & \ hfill 3 & \ hfill -1 \\ \ hfill 3 & \ hfill -2 & \ hfill -9 \ end {array} \ text {} | \ text {} \ begin {array} {r} \ hfill 8 \\ \ hfill -2 \\ \ hfill 9 \ end {array} \ right] [/ latex]

                            Затем мы выполняем операции со строками для получения формы «строка-эшелон».

                            [латекс] \ begin {array} {rrrrr} \ hfill -2 {R} _ {1} + {R} _ {2} = {R} _ {2} \ to \ left [\ begin {array} { rrrrrr} \ hfill 1 & \ hfill & \ hfill -1 & \ hfill & \ hfill 1 & \ hfill \\ \ hfill 0 & \ hfill & \ hfill 5 & \ hfill & \ hfill -3 & \ hfill \\ \ hfill 3 & \ hfill & \ hfill -2 & \ hfill & \ hfill -9 & \ hfill \ end {array} | \ begin {array} {rr} \ hfill & \ hfill 8 \\ \ hfill & \ hfill -18 \\ \ hfill & \ hfill 9 \ end {массив} \ right] & \ hfill & \ hfill & \ hfill & \ hfill -3 {R} _ {1} + {R} _ {3} = {R} _ {3} \ to \ left [\ begin {array} {rrrrrr} \ hfill 1 & \ hfill & \ hfill -1 & \ hfill & \ hfill 1 & \ hfill \\ \ hfill 0 & \ hfill & \ hfill 5 & \ hfill & \ hfill -3 & \ hfill \\ \ hfill 0 & \ hfill & \ hfill 1 & \ hfill & \ hfill -12 & \ hfill \ end {array} | \ begin {array} {rr} \ hfill & \ hfill 8 \\ \ hfill & \ hfill -18 \\ \ hfill & \ hfill -15 \ end {array} \ right] \ end {array} [/ latex]

                            Самый простой способ получить 1 в строке 2 столбца 1 — это поменять местами [латекс] {R} _ {2} [/ latex] и [latex] {R} _ {3} [/ latex].

                            [латекс] \ text {Interchange} {R} _ {2} \ text {и} {R} _ {3} \ to \ left [\ begin {array} {rrrrrrr} \ hfill 1 & \ hfill & \ hfill — 1 & \ hfill & \ hfill 1 & \ hfill & \ hfill 8 \\ \ hfill 0 & \ hfill & \ hfill 1 & \ hfill & \ hfill -12 & \ hfill & \ hfill -15 \\ \ hfill 0 & \ hfill & \ hfill 5 & \ hfill & \ hfill -3 & \ hfill & \ hfill -18 \ end {array} \ right] [/ latex]

                            Затем

                            [латекс] \ begin {array} {l} \\ \ begin {array} {rrrrr} \ hfill -5 {R} _ {2} + {R} _ {3} = {R} _ {3} \ в \ left [\ begin {array} {rrrrrr} \ hfill 1 & \ hfill & \ hfill -1 & \ hfill & \ hfill 1 & \ hfill \\ \ hfill 0 & \ hfill & \ hfill 1 & \ hfill & \ hfill -12 & \ hfill \\ \ hfill 0 & \ hfill & \ hfill 0 & \ hfill & \ hfill 57 & \ hfill \ end {array} | \ begin {array} {rr} \ hfill & \ hfill 8 \\ \ hfill & \ hfill -15 \\ \ hfill & \ hfill 57 \ end {array} \ right] & \ hfill & \ hfill & \ hfill & \ hfill — \ frac {1} {57} {R} _ {3} = {R} _ {3} \ to \ left [\ begin {array} {rrrrrr} \ hfill 1 & \ hfill & \ hfill -1 & \ hfill & \ hfill 1 & \ hfill \\ \ hfill 0 & \ hfill & \ hfill 1 & \ hfill & \ hfill -12 & \ hfill \\ \ hfill 0 & \ hfill & \ hfill 0 & \ hfill & \ hfill 1 & \ hfill \ end {array} | \ begin {array} {rr} \ hfill & \ hfill 8 \\ \ hfill & \ hfill -15 \ \ \ hfill & \ hfill 1 \ end {array} \ right] \ end {array} \ end {array} [/ latex]

                            Последняя матрица представляет собой эквивалентную систему.

                            [латекс] \ begin {массив} {l} \ text {} x-y + z = 8 \ hfill \\ \ text {} y — 12z = -15 \ hfill \\ \ text {} z = 1 \ hfill \ end {array} [/ latex]

                            Используя обратную подстановку, мы получаем решение как [latex] \ left (4, -3,1 \ right) [/ latex].

                            Напомним, что есть три возможных исхода решений для линейных систем. В предыдущем примере решение [латекс] \ left (4, -3,1 \ right) [/ latex] представляет точку в трехмерном пространстве. Эта точка представляет собой пересечение трех плоскостей.В следующем примере мы решаем систему, используя операции со строками, и обнаруживаем, что она представляет зависимую систему. Зависимая система в 3-х измерениях может быть представлена ​​двумя идентичными плоскостями, как в 2-х измерениях, где зависимая система представляет две идентичные линии.

                            Пример: решение зависимой системы 3 x 3

                            Решите следующую систему линейных уравнений, используя метод исключения Гаусса.

                            [латекс] \ begin {array} {r} \ hfill -x — 2y + z = -1 \\ \ hfill 2x + 3y = 2 \\ \ hfill y — 2z = 0 \ end {array} [/ latex]

                            Показать решение

                            Запишите расширенную матрицу.

                            [латекс] \ left [\ begin {array} {rrr} \ hfill -1 & \ hfill -2 & \ hfill 1 \\ \ hfill 2 & \ hfill 3 & \ hfill 0 \\ \ hfill 0 & \ hfill 1 & \ hfill -2 \ end {array} \ text {} | \ text {} \ begin {array} {r} \ hfill -1 \\ \ hfill 2 \\ \ hfill 0 \ end {array} \ right] [/ latex]

                            Сначала умножьте строку 1 на [latex] -1 [/ latex], чтобы получить 1 в строке 1, столбце 1. Затем выполните операции со строками , чтобы получить форму строки-эшелон.

                            [латекс] — {R} _ {1} \ to \ left [\ begin {array} {rrrrrrr} \ hfill 1 & \ hfill & \ hfill 2 & \ hfill & \ hfill -1 & \ hfill & \ hfill 1 \\ \ hfill 2 & \ hfill & \ hfill 3 & \ hfill & \ hfill 0 & \ hfill & \ hfill 2 \\ \ hfill 0 & \ hfill & \ hfill 1 & \ hfill & \ hfill -2 & \ hfill & \ hfill 0 \ end {array} \ справа] [/ латекс]

                            [латекс] {R} _ {2} \ leftrightarrow {R} _ {3} \ to \ left [\ begin {array} {rrrrr} \ hfill 1 & \ hfill & \ hfill 2 & \ hfill & \ hfill -1 \ \ \ hfill 0 & \ hfill & \ hfill 1 & \ hfill & \ hfill -2 \\ \ hfill 2 & \ hfill & \ hfill 3 & \ hfill & \ hfill 0 \ end {array} \ text {} | \ begin {array} { rr} \ hfill & \ hfill 1 \\ \ hfill & \ hfill 0 \\ \ hfill & \ hfill 2 \ end {array} \ right] [/ latex]

                            [латекс] -2 {R} _ {1} + {R} _ {3} = {R} _ {3} \ to \ left [\ begin {array} {rrrrrr} \ hfill 1 & \ hfill & \ hfill 2 & \ hfill & \ hfill -1 & \ hfill \\ \ hfill 0 & \ hfill & \ hfill 1 & \ hfill & \ hfill -2 & \ hfill \\ \ hfill 0 & \ hfill & \ hfill -1 & \ hfill & \ hfill 2 & \ hfill \ end {array} | \ begin {array} {rr} \ hfill & \ hfill 1 \\ \ hfill & \ hfill 0 \\ \ hfill & \ hfill 0 \ end {array} \ right] [/ latex]

                            [латекс] {R} _ {2} + {R} _ {3} = {R} _ {3} \ to \ left [\ begin {array} {rrrrrr} \ hfill 1 & \ hfill & \ hfill 2 & \ hfill & \ hfill -1 & \ hfill \\ \ hfill 0 & \ hfill & \ hfill 1 & \ hfill & \ hfill -2 & \ hfill \\ \ hfill 0 & \ hfill & \ hfill 0 & \ hfill & \ hfill 0 & \ hfill \ end { array} | \ begin {array} {rr} \ hfill & \ hfill 2 \\ \ hfill & \ hfill 1 \\ \ hfill & \ hfill 0 \ end {array} \ right] [/ latex]

                            Последняя матрица представляет следующую систему.

                            [латекс] \ begin {массив} {l} \ text {} x + 2y-z = 1 \ hfill \\ \ text {} y — 2z = 0 \ hfill \\ \ text {} 0 = 0 \ hfill \ конец {array} [/ latex]

                            По тождеству [latex] 0 = 0 [/ latex] мы видим, что это зависимая система с бесконечным числом решений. Затем мы находим общее решение. Решив второе уравнение для [latex] y [/ latex] и подставив его в первое уравнение, мы можем решить для [latex] z [/ latex] через [latex] x [/ latex].

                            [латекс] \ begin {array} {l} \ text {} x + 2y-z = 1 \ hfill \\ \ text {} y = 2z \ hfill \\ \ hfill \\ x + 2 \ left (2z \ справа) -z = 1 \ hfill \\ \ text {} x + 3z = 1 \ hfill \\ \ text {} z = \ frac {1-x} {3} \ hfill \ end {array} [/ latex]

                            Теперь мы подставляем выражение для [latex] z [/ latex] во второе уравнение, чтобы решить для [latex] y [/ latex] через [latex] x [/ latex].

                            [латекс] \ begin {массив} {l} \ text {} y — 2z = 0 \ hfill \\ \ text {} z = \ frac {1-x} {3} \ hfill \\ \ hfill \\ y — 2 \ left (\ frac {1-x} {3} \ right) = 0 \ hfill \\ \ text {} y = \ frac {2 — 2x} {3} \ hfill \ end {array} [/ latex ]

                            Общее решение — [latex] \ left (x, \ frac {2 — 2x} {3}, \ frac {1-x} {3} \ right) [/ latex].

                            Общее решение для зависимой системы 3 X 3

                            Напомним, что когда вы решаете зависимую систему линейных уравнений с двумя переменными с использованием исключения или подстановки, вы можете записать решение [latex] (x, y) [/ latex] через x, потому что существует бесконечно много (x, y) пары, которые будут удовлетворять зависимой системе уравнений, и все они попадают на линию [латекс] (x, mx + b) [/ latex].Теперь, когда вы работаете в трех измерениях, решение будет представлять собой плоскость, поэтому вы должны записать его в общей форме [латекс] (x, m_ {1} x + b_ {1}, m_ {2} x + b_ { 2}) [/ латекс].

                            Попробуй

                            Решите систему методом исключения Гаусса.

                            [латекс] \ begin {array} {c} x + 4y-z = 4 \\ 2x + 5y + 8z = 15 \ x + 3y — 3z = 1 \ end {array} [/ latex]

                            Показать решение

                            [латекс] \ левый (1,1,1 \ правый) [/ латекс]

                            Вопросы и ответы

                            Можно ли решить любую систему линейных уравнений методом исключения Гаусса?

                            Да, система линейных уравнений любого размера может быть решена методом исключения Гаусса.

                            Как: решить систему уравнений с помощью матриц с помощью калькулятора

                            1. Сохраните расширенную матрицу как матричную переменную [latex] \ left [A \ right], \ left [B \ right], \ left [C \ right] \ text {,} \ dots [/ latex].
                            2. Используйте функцию ref ( в калькуляторе, вызывая каждую матричную переменную по мере необходимости.

                            Пример: решение систем уравнений с помощью калькулятора

                            Решите систему уравнений.

                            [латекс] \ begin {array} {r} \ hfill 5x + 3y + 9z = -1 \\ \ hfill -2x + 3y-z = -2 \\ \ hfill -x — 4y + 5z = 1 \ end { array} [/ latex]

                            Показать решение

                            Напишите расширенную матрицу для системы уравнений.

                            [латекс] \ left [\ begin {array} {rrr} \ hfill 5 & \ hfill 3 & \ hfill 9 \\ \ hfill -2 & \ hfill 3 & \ hfill -1 \\ \ hfill -1 & \ hfill -4 & \ hfill 5 \ end {array} \ text {} | \ text {} \ begin {array} {r} \ hfill -1 \\ \ hfill -2 \\ \ hfill 1 \ end {array} \ right] [/ latex]

                            На странице матриц калькулятора введите расширенную матрицу выше как матричную переменную [latex] \ left [A \ right] [/ latex].

                            [латекс] \ left [A \ right] = \ left [\ begin {array} {rrrrrrr} \ hfill 5 & \ hfill & \ hfill 3 & \ hfill & \ hfill 9 & \ hfill & \ hfill -1 \\ \ hfill — 2 & \ hfill & \ hfill 3 & \ hfill & \ hfill -1 & \ hfill & \ hfill -2 \\ \ hfill -1 & \ hfill & \ hfill -4 & \ hfill & \ hfill 5 & \ hfill & \ hfill 1 \ end {массив } \ right] [/ latex]

                            Используйте функцию ref ( в калькуляторе, вызывая матричную переменную [latex] \ left [A \ right] [/ latex].

                            [латекс] \ text {ref} \ left (\ left [A \ right] \ right) [/ латекс]

                            Оценить.

                            [латекс] \ begin {array} {l} \ hfill \\ \ left [\ begin {array} {rrrr} \ hfill 1 & \ hfill \ frac {3} {5} & \ hfill \ frac {9} {5 } & \ hfill \ frac {1} {5} \\ \ hfill 0 & \ hfill 1 & \ hfill \ frac {13} {21} & \ hfill — \ frac {4} {7} \\ \ hfill 0 & \ hfill 0 & \ hfill 1 & \ hfill — \ frac {24} {187} \ end {array} \ right] \ to \ begin {array} {l} x + \ frac {3} {5} y + \ frac {9} {5} z = — \ frac {1} {5} \ hfill \\ \ text {} y + \ frac {13} {21} z = — \ frac {4} {7} \ hfill \\ \ text {} z = — \ frac {24} {187} \ hfill \ end {array} \ hfill \ end {array} [/ latex]

                            При использовании обратной подстановки решение: [latex] \ left (\ frac {61} {187}, — \ frac {92} {187}, — \ frac {24} {187} \ right) [/ latex] .

                            Приложения систем уравнений

                            Теперь обратимся к приложениям, для которых используются системы уравнений. В следующем примере мы определяем, сколько денег было инвестировано по двум разным ставкам, учитывая сумму процентов, полученных на обоих счетах.

                            Пример: применение матриц 2 × 2 к финансам

                            Кэролайн инвестирует в общей сложности 12 000 долларов в две муниципальные облигации, одна из которых выплачивает 10,5% годовых, а другая — 12%. Годовой процент, полученный по двум инвестициям в прошлом году, составил 1335 долларов.Сколько было вложено по каждой ставке?

                            Показать решение

                            У нас есть система двух уравнений с двумя переменными. Пусть [latex] x = [/ latex] сумма, инвестированная под 10,5% годовых, и [latex] y = [/ latex] сумма, инвестированная под 12% годовых.

                            [латекс] \ begin {array} {l} \ text {} x + y = 12 000 \ hfill \\ 0.105x + 0.12y = 1335 \ hfill \ end {array} [/ latex]

                            В качестве матрицы имеем

                            [латекс] \ left [\ begin {array} {rr} \ hfill 1 & \ hfill 1 \\ \ hfill 0.105 & \ hfill 0.12 \ end {array} \ text {} | \ text {} \ begin {array} { r} \ hfill 12 000 \\ \ hfill 1,335 \ end {array} \ right] [/ latex]

                            Умножить строку 1 на [латекс] -0.105 [/ latex] и добавьте результат в строку 2.

                            [латекс] \ left [\ begin {array} {rr} \ hfill 1 & \ hfill 1 \\ \ hfill 0 & \ hfill 0.015 \ end {array} \ text {} | \ text {} \ begin {array} {r } \ hfill 12,000 \\ \ hfill 75 \ end {array} \ right] [/ latex]

                            Затем,

                            [латекс] \ begin {array} {l} 0,015y = 75 \ hfill \\ \ text {} y = 5,000 \ hfill \ end {array} [/ latex]

                            Итак [латекс] 12 000 — 5 000 = 7 000 [/ латекс].

                            Таким образом, 5000 долларов были инвестированы под 12% годовых и 7000 долларов под 10,5%.

                            Пример: применение матриц 3 × 3 к финансам

                            Ava инвестирует в общей сложности 10 000 долларов в три счета, один из которых платит 5%, другой — 8%, а третий — 9%.Годовой процент, полученный по трем инвестициям в прошлом году, составил 770 долларов. Сумма, вложенная под 9%, была вдвое больше, чем сумма, вложенная под 5%. Сколько было вложено по каждой ставке?

                            Показать решение

                            У нас есть система трех уравнений с тремя переменными. Пусть [latex] x [/ latex] будет сумма, инвестированная под 5% годовых, пусть [latex] y [/ latex] будет суммой, инвестированной под 8%, и пусть [latex] z [/ latex] будет инвестированной суммой. под 9% годовых. Таким образом,

                            [латекс] \ begin {array} {l} \ text {} x + y + z = 10 000 \ hfill \\ 0.05x + 0,08y + 0,09z = 770 \ hfill \\ \ text {} 2x-z = 0 \ hfill \ end {array} [/ latex]

                            В качестве матрицы имеем

                            [латекс] \ left [\ begin {array} {rrr} \ hfill 1 & \ hfill 1 & \ hfill 1 \\ \ hfill 0,05 & \ hfill 0,08 & \ hfill 0,09 \\ \ hfill 2 & \ hfill 0 & \ hfill -1 \ end {array} \ text {} | \ text {} \ begin {array} {r} \ hfill 10,000 \\ \ hfill 770 \\ \ hfill 0 \ end {array} \ right] [/ latex]

                            Теперь мы выполняем исключение Гаусса, чтобы получить форму строки-эшелон.

                            [латекс] \ begin {массив} {l} \ begin {array} {l} \ hfill \\ -0.05 {R} _ {1} + {R} _ {2} = {R} _ {2} \ to \ left [\ begin {array} {rrrrrr} \ hfill 1 & \ hfill & \ hfill 1 & \ hfill & \ hfill 1 & \ hfill \\ \ hfill 0 & \ hfill & \ hfill 0.03 & \ hfill & \ hfill 0.04 & \ hfill \\ \ hfill 2 & \ hfill & \ hfill 0 & \ hfill & \ hfill -1 & \ hfill \ end {array} | \ begin {array} {rr} \ hfill & \ hfill 10,000 \\ \ hfill & \ hfill 270 \\ \ hfill & \ hfill 0 \ end {array} \ right] \ hfill \ end {array} \ hfill \\ -2 {R} _ {1} + {R} _ {3} = {R} _ {3} \ to \ left [\ begin {array} {rrrrrr} \ hfill 1 & \ hfill & \ hfill 1 & \ hfill & \ hfill 1 & \ hfill \\ \ hfill 0 & \ hfill & \ hfill 0.03 & \ hfill & \ hfill 0.04 & \ hfill \\ \ hfill 0 & \ hfill & \ hfill -2 & \ hfill & \ hfill -3 & \ hfill \ end {array} | \ begin {array} {rr} \ hfill & \ hfill 10,000 \\ \ hfill & \ hfill 270 \\ \ hfill & \ hfill -20,000 \ end {array} \ right] \ hfill \\ \ frac {1} {0.03} {R} _ {2} = {R} _ {2} \ to \ left [\ begin {array} {rrrrrr} \ hfill 0 & \ hfill & \ hfill 1 & \ hfill & \ hfill 1 & \ hfill \\ \ hfill 0 & \ hfill & \ hfill 1 & \ hfill & \ hfill \ frac {4} {3} & \ hfill \\ \ hfill 0 & \ hfill & \ hfill -2 & \ hfill & \ hfill -3 & \ hfill \ end {array} | \ begin {array} {rr} \ hfill & \ hfill 10,000 \\ \ hfill & \ hfill 9,000 \\ \ hfill & \ hfill -20,000 \ end {array} \ right] \ hfill \\ 2 {R} _ {2} + {R} _ {3} = {R} _ {3} \ to \ left [\ begin {array} {rrrrrr} \ hfill 1 & \ hfill & \ hfill 1 & \ hfill & \ hfill 1 & \ hfill \\ \ hfill 0 & \ hfill & \ hfill 1 & \ hfill & \ hfill \ frac {4} {3} & \ hfill \\ \ hfill 0 & \ hfill & \ hfill 0 & \ hfill & \ hfill — \ frac {1} {3} & \ hfill \ end {array} | \ begin {array} {rr} \ hfill & \ hfill 10,000 \\ \ hfill & \ hfill 9,000 \\ \ hfill & \ hfill -2,000 \ end {array} \ right] \ hfill \ end {array} [/ latex]

                            Третья строка сообщает нам [латекс] — \ frac {1} {3} z = -2,000 [/ latex]; таким образом [латекс] z = 6,000 [/ латекс].

                            Вторая строка сообщает нам [латекс] y + \ frac {4} {3} z = 9000 [/ latex].

                            Подставляя [латекс] z = 6,000 [/ латекс], получаем

                            [латекс] \ begin {array} {r} \ hfill y + \ frac {4} {3} \ left (6000 \ right) = 9000 \\ \ hfill y + 8000 = 9000 \\ \ hfill y = 1000 \ end {array} [/ latex]

                            Первая строка сообщает нам [латекс] x + y + z = 10,000 [/ latex]. Подставляя [latex] y = 1,000 [/ latex] и [latex] z = 6,000 [/ latex], мы получаем
                            [latex] \ begin {array} {l} x + 1,000 + 6,000 = 10,000 \ hfill \\ \ text {} x = 3 000 \ text {} \ hfill \ end {array} [/ latex]

                            Ответ: 3000 долларов вложены под 5%, 1000 долларов вложены под 8% и 6000 долларов вложены под 9%.

                            Попробуй

                            Небольшая обувная компания взяла ссуду в размере 1 500 000 долларов на расширение своего ассортимента. Часть денег была взята под 7%, часть — под 8%, часть — под 10%. Сумма займа под 10% в четыре раза превышала сумму займа под 7%, а годовая процентная ставка по всем трем займам составляла 130 500 долларов. Используйте матрицы, чтобы найти сумму займа по каждой ставке.

                            Показать решение

                            150 000 долларов США под 7%, 750 000 долларов США под 8%, 600 000 долларов США под 10%

                            Внесите свой вклад!

                            У вас была идея улучшить этот контент? Нам очень понравится ваш вклад.

                            Улучшить эту страницуПодробнее

                            ИСКЛЮЧЕНИЕ ПО ГАУССУ: РЕШЕНИЕ СИСТЕМ ЛИНЕЙНЫХ УРАВНЕНИЙ: ПРИМЕРЫ И РЕШАЕМЫЕ ЗАДАЧИ: СТАРШАЯ ШКОЛА

                            Содержимое этой страницы:


                            Введение

                            Система уравнений (линейная) — это группа (линейных) уравнений с
                            различные неизвестные факторы. Вообще говоря, неизвестные факторы появляются в различных уравнениях.

                            Уравнение с различными неизвестными факторами связывает их друг с другом.

                            Решение системы состоит в нахождении значения неизвестных факторов способом, который проверяет
                            все уравнения, составляющие систему.

                            • Если существует одно решение (одно значение для каждого неизвестного фактора), мы будем говорить, что
                              система — Согласованная независимая система (СНГ) .

                            • Если есть различные решения (система имеет бесконечно много решений), мы говорим, что система является
                              Согласованная зависимая система (CDS)..

                            • Если решения нет, а это произойдет, если есть два
                              или несколько уравнений, которые нельзя проверить одновременно,
                              мы говорим, что это несовместимая система (IS) . Например,
                              следующая система уравнений

                              $$ \ begin {case} \ begin {array} {lcl} y & = & 0 \\ 2x + y & = & 0 \\ 2x + y & = & 2 \ end {array} \ end {cases} $ $

                              несовместимо, потому что мы получаем решение x = 0 из второго уравнения и, из третьего, x = 1 .

                            В этом разделе мы собираемся решать системы, используя метод исключения Гаусса , который заключается в простом выполнении элементарных операций в строке или столбце расширенной матрицы, чтобы
                            получить свой эшелон из или его сокращенный эшелон
                            форма
                            (Гаусс-Иордан).


                            Метод разрешения

                            1. Применяем метод исключения Гаусса-Джордана : получаем
                              сокращенная форма эшелона строки из расширенной матрицы
                              систему уравнений, выполняя элементарные операции в строках (или столбцах).

                            2. Когда у нас есть матрица, мы применяем теорему Руше-Капелли , чтобы определить
                              тип системы и получить решение (я), а именно:

                              Пусть A · X = B будет системой из m линейных уравнений с n неизвестными
                              факторы, m и n натуральные числа (не ноль):

                              • AX = B соответствует тогда и только тогда, когда

                                $$ ранг (A) = ранг (A | B) $$

                              • AX = B непротиворечивый независимый тогда и только тогда, когда

                                $$ ранг (A) = n = ранг (A | B) $$

                            Примечание: Элементарные операции в строках или столбцах позволяют получить системы, эквивалентные исходной, но с формой, упрощающей получение решений (если они есть).Также есть более быстрые
                            инструменты для выработки решений в СНГ, такие как правило Крамера.




                            Система 1

                            Показать решение

                            Расширенная матрица системы

                            того же размера, что и система (2×3).
                            Вертикальная линия, отделяющая матричные коэффициенты от вектора независимых членов.

                            Выполняем элементарные операции в строках для получения приведенной формы эшелона строк:

                            Умножаем первую строку на 1/5 а вторую на 1/3

                            Добавляем вторую строку с первой

                            Вторую строку умножаем на 5/7

                            Складываем первую строку со второй, умноженной на -2/5

                            Эта последняя эквивалентная матрица представлена ​​в сокращенной форме эшелона строк.
                            и это позволяет нам быстро увидеть рейтинг
                            матрица коэффициентов и расширенная.

                            Рассчитываем ранги:

                            По теореме Руше-Капелли система непротиворечива Независимая. Полученная матрица представляет собой систему

                            , который является решением исходной системы.


                            Система 2

                            Показать решение

                            Расширенная матрица системы

                            Проводим элементарные операции в строках для получения приведенных
                            форма эшелона строки:

                            Вторую строку умножаем на 1/2

                            Складываем первую строку со второй

                            Умножаем первую строку на 1/3

                            Эта последняя эквивалентная матрица находится в форме сокращенного эшелона строк и имеет нулевую строку, что означает, что строки в исходной системе линейно зависимы.
                            (любой из них может быть получен путем умножения другого на скаляр, не равный нулю).

                            Рассчитываем ранги

                            По теореме Руше-Капелли система непротиворечива. Кроме того, это зависит, потому что ранг (1) ниже, чем количество неизвестных факторов (2).

                            Полученная матрица представляет собой систему

                            Решения


                            Система 3

                            Показать решение

                            Расширенная матрица системы

                            Выполняем элементарные операции в строках, чтобы получить сокращенную форму эшелона строк

                            Изменяем порядок строк (так у нас уже будет 1 в первой строке без необходимости умножать)

                            Складываем вторую строку с первой, умноженной на 5 :

                            Вторую строку умножаем на -1/15

                            Эта последняя матрица имеет эшелонированную форму (не сокращена).

                            Мы можем непосредственно заметить, что система несовместима, потому что у нас
                            следующее равенство (вторая строка):

                            $$ 0x + 0y = 1 $$

                            , это невозможное равенство.

                            Рассчитываем ранги матрицы коэффициентов и дополненной:

                            По теореме Руше-Капелли система несовместна (решения нет).
                            Полученная матрица представляет собой систему

                            Система непоследовательна из-за невозможного равенства

                            $$ 0 = 1 $$


                            Система 4

                            Показать решение

                            Расширенная матрица системы

                            (размер 3х4).

                            Выполняем элементарные операции в строках, чтобы получить сокращенную форму эшелона строк

                            Умножаем первую строку на 1/5

                            Складываем вторую и третью строки, умножив первую на -2

                            Вторую и третью строки умножаем на 5

                            Складываем вторую строку с третьей, умноженной на -1

                            Умножаем вторую строку на -1/10 , а третью на 1/11

                            Складываем первую строку со второй, умноженной на -2/5 , а третью со второй, умноженной на -1

                            Третью строку умножаем на -11/5

                            Эта последняя эквивалентная матрица находится в сокращенной форме эшелона строк (мы знаем ее, потому что это единичная матрица).Имея единичную матрицу, мы знаем, что это непротиворечивая независимая система, и можем получить единственное решение.

                            Рассчитываем ранги

                            По теореме Руше-Капелли система непротиворечива Независимая. Полученная матрица представляет собой систему

                            , который является решением системы.


                            Система 5

                            Показать решение

                            Расширенная матрица системы

                            Выполняем элементарные операции в строках, чтобы получить сокращенную форму эшелона строк

                            Мы вычитаем первую строку из второй и добавляем третью строку с первой.

                            Умножаем первую строку на 1/2 , а вторую на 1/3

                            Мы складываем первую строку со второй, умноженной на 1/2 , и третью строку со второй, умноженной на -2

                            Третью строку умножаем на 1/3

                            Складываем первую строку с третьей, умноженной на -3/2

                            Эта последняя матрица представляет собой эшелонированную матрицу с сокращенным числом строк (мы знаем это, потому что у нас есть единичная матрица).

                            Рассчитываем ранги

                            По теореме Руше-Капелли система непротиворечива Независимая. Полученная матрица представляет собой систему


                            Система 6

                            Показать решение

                            Расширенная матрица системы

                            Выполняем элементарные операции в строках, чтобы получить сокращенную форму эшелона строк

                            Умножаем вторую строку на -1/3 и третью на 1/4

                            Складываем вторую и третью строки, умножив первую на -1

                            Умножаем вторую строку на -3/4 и третью на -1

                            Третью строку складываем со второй, умноженной на -1

                            Эта последняя матрица имеет эшелонированную форму (но не сокращена)
                            и мы не продолжаем выполнять операции по строкам, потому что можем
                            видим, что последняя строка делает систему непоследовательной.Эта строка сообщает нам:

                            $$ 0x + 0y + 0z = -1 $$

                            И все это невозможное равенство.

                            Фактически у нас есть ранги

                            По теореме Руше-Капелли система несовместна.


                            Система 7

                            Показать решение

                            Расширенная матрица системы

                            Выполняем элементарные операции в строках, чтобы получить сокращенную форму эшелона строк

                            Умножаем первую и вторую строки на 1/3 и 1/6 соответственно

                            Вторую строку вычитаем из третьей

                            Складываем первую строку со второй

                            Третью строку умножаем на 3

                            Эта последняя матрица находится в сокращенном эшелоне строк формы , поэтому мы можем легко вычислить ранги:

                            Рассчитываем ранги

                            По теореме Руше-Капелли система непротиворечива.Но он не является независимым, потому что количество неизвестных факторов (3) отличается от ранга.
                            Полученная матрица представляет собой систему

                            Решения


                            Система 8

                            Показать решение

                            Расширенная матрица системы

                            Примечание: прежде, чем мы начнем, мы должны прокомментировать, что процедура
                            будет таким же, как и до сих пор.Но у нас есть корни
                            в матрице, а это значит, что операции в строках будут длинными
                            и утомительно. Эта задача не очень интересна в дидактическом смысле, помимо расчетов.

                            Проводим элементарные операции в строках, чтобы получить
                            редуцированный рядный эшелон формы

                            Мы складываем вторую строку с первой, умноженной на -√5 и, с
                            третья, умноженная на -2/5

                            Умножаем вторую строку на 1 / √5 и третью на 5/17

                            Складываем первую строку с третьей и со второй вычитаем третью

                            Переписываем матрицу

                            Умножаем вторую строку на (√5 / 5-5) -1

                            Складываем первую строку со второй, умноженной на -5

                            В этой последней матрице он (почти) в строке уменьшен
                            форма эшелона (надо поменять второй и третий ряды
                            так что это правда).Из последней матрицы получаем решения:


                            Система 9

                            Показать решение

                            Расширенная матрица системы

                            Выполняем элементарные операции в строках, чтобы получить сокращенную форму эшелона строк

                            Складываем третью и четвертую строки, умножая первую на 3 и -2 соответственно

                            Складываем первую, вторую и третью строки с четвертой, умноженной на 2 , -3 и 5 соответственно

                            Умножаем четвертую строку на -1 и меняем на вторую

                            Умножаем третью и четвертую строки на 1/34 и -1/22 соответственно

                            Вычитаем третью строку из четвертой

                            Четвертую умножаем на -187/42

                            Добавляем в первую строку третью умноженную на -13 и вторую на 8

                            К первой строке прибавляем четвертую, умноженную на 5/34 , ко второй прибавляем первую, умноженную на
                            5/17 и к третьему добавляем первое, умноженное на -3/34

                            По теореме Руше-Капелли система является непротиворечивой Независимой и решением является


                            Система 10

                            Показать решение

                            Расширенная матрица системы

                            Примечание: Эта система была включена в
                            цель отметить, что
                            теория матрицы применима к комплексным числам.Единственное отличие от предыдущих систем в том, что
                            теперь нам нужно действовать путем умножения и деления
                            комплексные числа.

                            Выполняем элементарные операции в строках для получения приведенной формы эшелона строк

                            Умножаем вторую строку на 1/2 и меняем ее на первую

                            Добавляем вторую строку с первой, умноженной на — (1 + i)

                            Вторую строку умножаем на

                            Добавляем первую строку ко второй умноженной на -i / 2

                            Эта последняя матрица имеет сокращенную форму, поскольку она является единичной матрицей.

                            По теореме Руше-Капелли система является непротиворечивой Независимой и решением является


                            Matesfacil.com
                            компании J. Llopis под лицензией
                            творческий
                            Международная лицензия Commons Attribution-NonCommercial 4.0.

                            Системы линейных уравнений: исключение Гаусса

                            Решение линейной системы с матрицами с использованием исключения Гаусса

                            После нескольких уроков, в которых мы неоднократно упоминали, что мы охватываем основы, необходимые для последующего изучения того, как решать системы линейных уравнений, пришло время для нашего урока сосредоточиться на полной методологии, которой нужно следовать, чтобы найти решения. для таких систем.

                            Что такое гауссово исключение

                            Исключение Гаусса — это название метода, который мы используем для выполнения трех типов операций со строками матрицы над расширенной матрицей, полученной из линейной системы уравнений, чтобы найти решения для такой системы. Этот метод также называется сокращением строк и состоит из двух этапов: прямого исключения и обратной замены.

                            Эти два шага метода исключения Гаусса различаются не операциями, которые вы можете использовать с их помощью, а результатом, который они производят.Шаг прямого исключения относится к сокращению строки, необходимому для упрощения рассматриваемой матрицы до ее эшелонированной формы. Такой этап имеет целью продемонстрировать, имеет ли система уравнений, изображенная в матрице, единственное возможное решение, бесконечное множество решений или просто отсутствие решения. Если обнаружено, что система не имеет решения, то нет причин продолжать сокращение строки матрицы на следующем этапе.

                            Если возможно получить решения для переменных, входящих в линейную систему, то выполняется этап исключения Гаусса с обратной подстановкой.На этом последнем шаге будет получена сокращенная форма матрицы, которая, в свою очередь, дает общее решение системы линейных уравнений.

                            Правила исключения Гаусса такие же, как правила для трех элементарных операций со строками, другими словами, вы можете алгебраически оперировать строками матрицы следующими тремя способами (или комбинацией):

                            1. Перестановка двух рядов
                            2. Умножение строки на константу (любую константу, отличную от нуля)
                            3. Добавление строки к другой строке

                            Итак, решение линейной системы с матрицами с использованием исключения Гаусса оказывается структурированным, организованным и довольно эффективным методом.

                            Как выполнить исключение по Гауссу

                            На самом деле это не установленный набор шагов исключения Гаусса, которым нужно следовать, чтобы решить систему линейных уравнений, это все о матрице, которая у вас есть в ваших руках, и необходимых операциях со строками для ее упрощения. Для этого давайте поработаем над нашим первым примером исключения Гаусса, чтобы вы могли начать изучать весь процесс и интуицию, которая необходима при работе с ними:

                            Пример 1

                            Обратите внимание, что в этот момент мы можем заметить, что эта система линейных уравнений разрешима с единственным решением для каждой из ее переменных.То, что мы выполнили до сих пор, — это первый этап сокращения строк: прямое исключение. Мы можем продолжить упрощение этой матрицы еще больше (что приведет нас ко второму этапу обратной подстановки), но нам это действительно не нужно, поскольку на этом этапе система легко разрешима. Таким образом, мы смотрим на получившуюся систему, чтобы решить ее напрямую:

                            • Уравнение 5: Полученная линейная система уравнений для решения

                            Из этого набора мы можем автоматически заметить, что значение переменной z равно: z = -2.Мы используем это знание, чтобы подставить его во вторые уравнения для решения относительно y, и подставить значения y и z в первые уравнения для решения относительно x:

                            В последний раздел этого урока добавлено больше задач исключения Гаусса. Обязательно проработайте их, чтобы практиковаться.

                            Разница между исключением по Гауссу и по Гауссу Иордану

                            Разница между методом исключения Гаусса и исключением Гаусса Жордана заключается в том, что один создает матрицу в форме эшелона строк, а другой — матрицу в форме уменьшенного ряда.Матрица формы эшелона строк имеет верхнюю треугольную композицию, где любые нулевые строки находятся внизу, а ведущие члены находятся справа от ведущего члена из строки выше. Уменьшенная форма эшелона выходит за рамки еще большего упрощения (иногда даже достигая формы единичной матрицы).

                            Уравнение 8: Разница между формой эшелона и формой ряда эшелонов

                            История исключения Гаусса и его названия довольно интересны, вы будете удивлены, узнав, что название «Гауссовский» было присвоено этой методологии по ошибке в прошлом веке.В действительности было обнаружено, что алгоритм одновременного решения системы линейных уравнений с использованием матриц и редукции строк записан в той или иной форме в древних китайских текстах, которые датируются еще до нашей эры. Затем в конце 1600-х годов Исаак Ньютон провел по этому уроку, чтобы заполнить то, что он считал пробелом в книгах по алгебре. После того, как название «Гауссиан» было уже установлено в 1950-х годах, термин Гаусса-Иордана был принят, когда геодезист У. Джордан усовершенствовал технику, чтобы он мог использовать такие вычисления для обработки своих наблюдаемых данных топографической съемки.Если вы хотите продолжить чтение увлекательной истории математиков исключения Гаусса, не бойтесь щелкнуть ссылку и прочитать.

                            На самом деле нет никакой физической разницы между исключением Гаусса и исключением Гаусса Джордана, оба процесса следуют одному и тому же типу операций со строками и их комбинациям, их различие зависит от результатов, которые они производят. Многие математики и учителя во всем мире будут относиться к исключению Гаусса и исключению Гаусса Джордана как к методам создания матрицы эшелонированной формы по сравнению с методом создания матрицы уменьшенной эшелонированной формы, но на самом деле они говорят о двух стадиях сокращения строк. мы объяснили это в самом первом разделе этого урока (прямое исключение и обратная подстановка), и поэтому вы просто применяете операции со строками, пока не упростите рассматриваемую матрицу.Если вы дойдете до формы эшелона, вы обычно можете решить с ней систему линейных уравнений (до сих пор это то, что называлось бы исключением Гаусса). Если вам нужно продолжить упрощение такой матрицы, чтобы напрямую получить общее решение для системы уравнений, над которой вы работаете, в этом случае вы просто продолжаете работать с матрицей по строкам, пока не упростите ее до сокращенной формы эшелона. (это будет то, что мы называем частью Гаусса-Жордана, и которую можно рассматривать также как поворотное исключение Гаусса).

                            Мы оставим подробное объяснение форм сокращения строк и эшелонирования для следующего урока, поскольку сейчас вам нужно знать, что, если у вас нет единичной матрицы в левой части расширенной матрицы, которую вы решаете (в этом случае вы не используете не нужно ничего делать для решения системы уравнений, относящейся к матрице), метод исключения Гаусса (регулярное сокращение строк) всегда будет использоваться для решения линейной системы уравнений, которая была записана в виде матрицы.

                            Примеры исключения Гаусса

                            В качестве последнего раздела давайте поработаем еще несколько упражнений по исключению Гаусса (сокращение строк), чтобы вы могли больше попрактиковаться в этой методологии.На протяжении многих будущих уроков этого курса линейной алгебры вы обнаружите, что сокращение строк является одним из самых важных инструментов при работе с матричными уравнениями. Поэтому убедитесь, что вы понимаете все этапы решения следующих проблем.

                            Пример 2

                            Пример 3

                            Мы знаем, что для этой системы мы получим расширенную матрицу с тремя строками (поскольку система содержит три уравнения) и тремя столбцами слева от вертикальной линии (поскольку есть три разных переменных).В этом случае мы перейдем непосредственно к сокращению строк, и поэтому первая матрица, которую вы увидите в этом процессе, — это та, которую вы получите, преобразовав систему линейных уравнений в расширенную матрицу.

                            • Уравнение 15: Строка, уменьшающая расширенную матрицу

                            Обратите внимание, как мы можем сразу сказать, что переменная z равна нулю для этой системы, поскольку третья строка результирующей матрицы показывает уравнение -9z = 0 . Мы используем это знание и проверяем вторую строку матрицы, которая предоставит уравнение 2y — 6z = 0 , подставив в это уравнение значение z = 0 \, получится y \, также равное нулю.Таким образом, мы, наконец, подставляем оба значения y и z \ в уравнение, которое получается из первой строки матрицы: x + 4y + 3z = 1 , поскольку и y , и z \ , равны нулю, то это дает нам x = 1 . Итак, окончательное решение этой системы уравнений выглядит следующим образом:

                            • Уравнение 16: Окончательное решение системы уравнений

                            Пример 4

                            Из чего видно, что последняя строка дает уравнение: 6z = 3 и, следовательно, z = 1/2.Мы подставляем это в уравнения, полученные во второй и первой строках (в указанном порядке), чтобы вычислить значения переменных x и y:

                            Пример 5

                            • Решите следующую линейную систему, используя метод исключения Гаусса:

                              Уравнение 21: Система линейных уравнений с двумя переменными

                            • Транскрипция линейной системы в виде расширенной матрицы и редукции строк:

                              Уравнение 22: Строка, уменьшающая расширенную матрицу

                            • Что автоматически говорит нам y = 8 .Итак, подставляя это значение в уравнение из первой строки, получаем: 4x — 5y = 4x — 5 (8) = 4x — 40 = -6 4x = 34 \, и поэтому значение x равно: x = 172 \ frac {\ small17} {\ small2} 217 . И окончательное решение этой системы уравнений:

                              Уравнение 23: Окончательное решение системы уравнений

                            Пример 6

                            Чтобы завершить наш урок на сегодня, у нас есть рекомендация по ссылке для дополнения ваших исследований: Исключение Гаусса — статья, которая содержит некоторую дополнительную информацию о сокращении строк, включая введение в тему и еще несколько примеров.Как мы упоминали ранее, будьте готовы продолжать использовать сокращение строк почти на протяжении всего курса линейной алгебры, так что до встречи на следующем уроке!

                            Решение систем с исключением Гаусса — College Algebra

                            Цели обучения

                            В этом разделе вы:

                            • Напишите расширенную матрицу системы уравнений.
                            • Напишите систему уравнений из расширенной матрицы.
                            • Выполнение операций со строками в матрице.
                            • Решите систему линейных уравнений, используя матрицы.

                            Немецкий математик Карл Фридрих Гаусс (1777–1855).

                            Карл Фридрих Гаусс жил в конце 18-го и начале 19-го веков, но он по-прежнему считается одним из самых плодовитых математиков в истории. Его вклад в математику и физику охватывает такие области, как алгебра, теория чисел, анализ, дифференциальная геометрия, астрономия и оптика.Его открытия в области теории матриц изменили способ работы математиков за последние два столетия.

                            Мы впервые столкнулись с методом исключения Гаусса в системах линейных уравнений: две переменные. В этом разделе мы еще раз вернемся к этой технике решения систем, на этот раз с использованием матриц.

                            Написание расширенной матрицы системы уравнений

                            Матрица может служить средством представления и решения системы уравнений. Чтобы выразить систему в матричной форме, мы извлекаем коэффициенты переменных и констант, и они становятся элементами матрицы.Мы используем вертикальную линию, чтобы отделить записи коэффициентов от констант, по сути заменяя знаки равенства. Когда система написана в такой форме, мы называем ее расширенной матрицей.

                            Например, рассмотрим следующую систему уравнений.

                            Мы можем записать эту систему в виде расширенной матрицы:

                            Мы также можем написать матрицу, содержащую только коэффициенты. Это называется матрицей коэффициентов.

                            Система уравнений три на три, например

                            имеет матрицу коэффициентов

                            и представлена ​​расширенной матрицей

                            Обратите внимание, что матрица написана так, что переменные выстраиваются в свои собственные столбцы: элементы x идут в первый столбец, элементы y — во второй столбец и z — элементы в третьем столбце.Очень важно, чтобы каждое уравнение было написано в стандартной форме, чтобы переменные совпадали. Если в уравнении отсутствует член переменной, коэффициент равен 0.

                            Для данной системы уравнений напишите расширенную матрицу.

                            1. Запишите коэффициенты членов x как числа в первом столбце.
                            2. Запишите коэффициенты членов y как числа во втором столбце.
                            3. Если имеется z -термов, запишите коэффициенты в виде чисел в третьем столбце.
                            4. Нарисуйте вертикальную линию и напишите константы справа от нее.

                            Написание расширенной матрицы для системы уравнений

                            Напишите расширенную матрицу для данной системы уравнений.

                            Расширенная матрица отображает коэффициенты переменных и дополнительный столбец для констант.

                            Запишите расширенную матрицу данной системы уравнений.

                            Написание системы уравнений из расширенной матрицы

                            Мы можем использовать расширенные матрицы, чтобы помочь нам решать системы уравнений, потому что они упрощают операции, когда системы не обременены переменными.Однако важно понимать, как переключаться между форматами, чтобы поиск решений был более плавным и интуитивно понятным. Здесь мы будем использовать информацию в расширенной матрице, чтобы записать систему уравнений в стандартной форме.

                            Напишите систему уравнений из расширенной матрицы.

                            Выполнение операций со строками в матрице

                            Теперь, когда мы можем писать системы уравнений в форме расширенной матрицы, мы рассмотрим различные операции со строками, которые могут выполняться с матрицей, такие как сложение, умножение на константу и перестановка строк.

                            Выполнение строковых операций над матрицей — это метод, который мы используем для решения системы уравнений. Чтобы решить систему уравнений, мы хотим преобразовать матрицу в форму строки-эшелона, в которой есть единицы вниз по главной диагонали от верхнего левого угла до нижнего правого угла и нули в каждой позиции ниже главной диагонали. как показано.

                            Мы используем операции со строками, соответствующие операциям с уравнениями, чтобы получить новую матрицу, эквивалентную строкам в более простой форме.Вот рекомендации по получению формы рядного эшелона.

                            1. В любой ненулевой строке первым ненулевым числом является 1. Оно называется ведущим 1.
                            2. Любые нулевые строки помещаются внизу матрицы.
                            3. Любая ведущая 1 находится ниже и правее предыдущей ведущей 1.
                            4. Любой столбец, в котором в начале стоит 1, имеет нули во всех остальных позициях в столбце.

                            Чтобы решить систему уравнений, мы можем выполнить следующие операции со строками, чтобы преобразовать матрицу коэффициентов в форму ряда строк и выполнить обратную подстановку, чтобы найти решение.

                            1. Поменять местами ряды. (Обозначение 🙂
                            2. Умножить строку на константу. (Обозначение 🙂
                            3. Добавить произведение одной строки на константу к другой строке. (Замечание:

                            Каждая из строковых операций соответствует операциям, которые мы уже научились решать системы уравнений с тремя переменными. С помощью этих операций есть несколько ключевых шагов, которые быстро достигнут цели написания матрицы в виде эшелона строк. Чтобы получить матрицу в виде эшелона строк для поиска решений, мы используем метод исключения Гаусса, который использует операции со строками для получения 1 в качестве первой записи, чтобы строку 1 можно было использовать для преобразования оставшихся строк.

                            Исключение по Гауссу

                            Метод исключения Гаусса относится к стратегии, используемой для получения матрицы в виде строки-эшелона. Цель состоит в том, чтобы записать матрицу с номером 1 в качестве записи по главной диагонали и иметь все нули внизу.

                            Первый шаг стратегии Гаусса включает получение 1 в качестве первой записи, так что строка 1 может использоваться для изменения строк ниже.

                            Учитывая расширенную матрицу, выполните операции со строками для получения формы «строка-эшелон».

                            1. Первое уравнение должно иметь старший коэффициент 1. При необходимости поменяйте местами строки или умножьте на константу.
                            2. Используйте операции со строками, чтобы получить нули в первом столбце под первой записью 1.
                            3. Используйте операции со строками, чтобы получить 1 в строке 2, столбце 2.
                            4. Используйте операции со строками, чтобы получить нули в нижнем столбце 2, ниже записи 1.
                            5. Используйте операции со строками, чтобы получить 1 в строке 3, столбце 3.
                            6. Продолжайте этот процесс для всех строк, пока в каждой записи по главной диагонали не будет 1, а внизу будут только нули.
                            7. Если какие-либо строки содержат все нули, поместите их внизу.

                            Решение системы методом исключения Гаусса

                            Решите данную систему методом исключения Гаусса.

                            Решите данную систему методом исключения Гаусса.

                            Использование исключения Гаусса для решения системы уравнений

                            Используйте метод исключения Гаусса для решения данной системы уравнений.

                            Решение зависимой системы

                            Решите систему уравнений.

                            Выполнение операций со строками в расширенной матрице 3×3 для получения формы Row-Echelon

                            Выполняет строковые операции с заданной матрицей для получения формы «строка-эшелон».

                            Запишите систему уравнений в виде строк.

                            Решение системы линейных уравнений с использованием матриц

                            Мы видели, как написать систему уравнений с расширенной матрицей, а затем как использовать строковые операции и обратную подстановку для получения строчно-эшелонированной формы.Теперь мы перейдем на шаг дальше от строковой формы, чтобы решить систему линейных уравнений 3 на 3. Общая идея состоит в том, чтобы исключить все переменные, кроме одной, с помощью операций со строками, а затем выполнить обратную замену для поиска других переменных.

                            Решение системы линейных уравнений с использованием матриц

                            Решите систему линейных уравнений с помощью матриц.

                            Решение зависимой системы линейных уравнений с использованием матриц

                            Решите следующую систему линейных уравнений, используя матрицы.

                            Решите систему, используя матрицы.

                            Можно ли решить любую систему линейных уравнений методом исключения Гаусса?

                            Да, система линейных уравнений любого размера может быть решена методом исключения Гаусса.

                            Дана система уравнений, решите с помощью матриц с помощью калькулятора.

                            1. Сохранить расширенную матрицу как матричную переменную
                            2. Используйте функцию ref ( в калькуляторе, вызывая каждую матричную переменную по мере необходимости.

                            Решение систем уравнений с матрицами с помощью калькулятора

                            Решите систему уравнений.

                            Применение матриц 2 × 2 к финансам

                            Кэролайн инвестирует в общей сложности 12 000 фунтов стерлингов в две муниципальные облигации, одна из которых выплачивает 10,5% годовых, а другая — 12%. Годовой процент, полученный по двум инвестициям в прошлом году, составил 1335 евро. Сколько было вложено по каждой ставке?

                            Применение матриц 3 × 3 к финансам

                            Ava инвестирует в общей сложности 10 000 фунтов стерлингов в три счета, один из которых платит 5% годовых, другой — 8%, а третий — 9%.Годовой процент, полученный по трем инвестициям в прошлом году, составил 770 фунтов стерлингов. Сумма, вложенная под 9%, была вдвое больше, чем сумма, вложенная под 5%. Сколько было вложено по каждой ставке?

                            У нас есть система трех уравнений с тремя переменными. Пусть будет сумма, вложенная под 5%, пусть будет сумма, вложенная под 8%, пусть будет сумма, вложенная под 9%. Таким образом,

                            В качестве матрицы имеем

                            Теперь мы выполняем исключение Гаусса, чтобы получить форму строки-эшелон.

                            Третья строка сообщает usthus

                            Вторая строка сообщает нам Подставляя, мы получаем

                            Первая строка говорит нам, что подставляем и получаем

                            Ответ: 3000 евро вложены под 5%, 1000 евро вложены под 8% и 6000 евро вложены под 9%.

                            Небольшая обувная компания взяла ссуду в размере 1 500 000 фунтов стерлингов для расширения своих запасов. Часть денег была взята под 7%, часть — под 8%, часть — под 10%. Сумма займа под 10% в четыре раза превышала сумму займа под 7%, а годовая процентная ставка по всем трем займам составляла 130 500 фунтов стерлингов. Используйте матрицы, чтобы найти сумму займа по каждой ставке.

                            — 150 000 фунтов стерлингов при 7%, 750 000 фунтов стерлингов при 8%, 600 000 фунтов стерлингов при 10%

                            Ключевые понятия

                            • Расширенная матрица — это матрица, которая содержит коэффициенты и константы системы уравнений.См. (Рисунок).
                            • Матрица, дополненная постоянным столбцом, может быть представлена ​​как исходная система уравнений. См. (Рисунок).
                            • Операции со строками включают в себя умножение строки на константу, добавление одной строки к другой строке и замену строк местами.
                            • Мы можем использовать метод исключения Гаусса для решения системы уравнений. См. (Рисунок), (Рисунок) и (Рисунок).
                            • Операции со строками выполняются над матрицами для получения формы «строка-эшелон». См. (Рисунок).
                            • Чтобы решить систему уравнений, запишите ее в форме дополненной матрицы.Выполните операции со строками, чтобы получить форму эшелона строк. Обратно-заменитель, чтобы найти решения. См. (Рисунок) и (Рисунок).
                            • Калькулятор можно использовать для решения систем уравнений с использованием матриц. См. (Рисунок).
                            • Многие реальные проблемы можно решить с помощью расширенных матриц. См. (Рисунок) и (Рисунок).

                            Упражнения по разделам

                            Устный

                            Можно ли записать любую систему линейных уравнений в виде расширенной матрицы? Объясните, почему да или почему нет. Объясните, как написать эту расширенную матрицу.

                            Да. Для каждой строки коэффициенты переменных записываются поперек соответствующей строки и помещается вертикальная черта; затем константы помещаются справа от вертикальной полосы.

                            Можно ли записать любую матрицу в виде системы линейных уравнений? Объясните, почему да или почему нет. Объясните, как написать эту систему уравнений.

                            Есть только один правильный метод использования операций со строками в матрице? Попытайтесь объяснить две различные операции со строками, которые можно выполнить для расширенной матрицы

                            .

                            Нет, существует множество правильных методов использования строковых операций над матрицей.Есть два возможных способа: (1) Поменять местами строки 1 и 2. Затем (2) Разделить строку 1 на 9.

                            Можно ли решить матрицу с нулевым элементом на диагонали? Объясните, почему да или почему нет. Что бы вы сделали, чтобы исправить ситуацию?

                            Может ли матрица с 0 элементами для всей строки иметь одно решение? Объясните, почему да или почему нет.

                            Нет. Матрица с 0 элементами для всей строки будет иметь либо ноль, либо бесконечно много решений.

                            Алгебраический

                            Для следующих упражнений напишите расширенную матрицу для линейной системы.

                            Для следующих упражнений запишите линейную систему из расширенной матрицы.

                            Для следующих упражнений решите систему методом исключения Гаусса.

                            Добавочные номера

                            Для следующих упражнений используйте метод исключения Гаусса для решения системы.

                            Реальные приложения

                            Для следующих упражнений настройте расширенную матрицу, описывающую ситуацию, и найдите желаемое решение.

                            Ежедневно в магазине кексов продается 5 000 кексов со вкусом шоколада и ванили. Если вкус шоколада в 3 раза популярнее, чем аромат ванили, сколько кексов продается в день?

                            В конкурирующем магазине кексов ежедневно продаются кексы на сумму 4520 фунтов стерлингов.Шоколадные кексы стоят 2,25 евро, а кексы из красного бархата — 1,75 евро. Если общее количество кексов, проданных в день, составляет 2200, сколько штук каждого вкуса продается каждый день?

                            860 красный бархат, 1340 шоколадный

                            Вы вложили 10 000 евро в два счета: один с простой процентной ставкой 3%, а другой — с процентной ставкой 2,5%. Если ваша общая сумма процентов по истечении одного года составила 283,50 фунтов стерлингов, какая сумма была на каждом счете по истечении года?

                            Вы вложили 2300 евро на счет 1 и 2700 евро на счет 2.Если общая сумма процентов по истечении одного года составляет 254 евро, а на счете 2 процентная ставка в 1,5 раза выше, чем на счете 1, каковы процентные ставки? Предположим простые процентные ставки.

                            4% на счет 1, 6% на счет 2

                            Bikes’R’Us производит велосипеды по 250 фунтов стерлингов. Производитель обошелся в 180 фунтов стерлингов за велосипед плюс стартовый взнос в размере 3500 фунтов стерлингов. Через сколько проданных велосипедов производитель выйдет на уровень безубыточности?

                            Крупный магазин бытовой техники рассматривает возможность приобретения пылесосов у небольшого производителя.Магазин сможет приобрести пылесосы по 86 фунтов стерлингов каждый, со стоимостью доставки 9 200 фунтов стерлингов, независимо от того, сколько пылесосов продано. Если магазин должен начать получать прибыль после продажи 230 единиц, сколько они должны взимать за пылесосы?

                            Три самых популярных вкуса мороженого — это шоколад, клубника и ваниль, составляющие 83% вкусов, продаваемых в магазине мороженого. Если ваниль продается на 1% больше, чем в два раза больше клубники, а шоколад продается на 11% больше, чем ваниль, сколько в общем потреблении мороженого приходится на ароматы ванили, шоколада и клубники?

                            В магазине мороженого возрастает спрос на три вкуса.В прошлом году банановое, тыквенное и мороженое с каменистой дорогой составили 12% от общего объема продаж мороженого. В этом году на те же три вида мороженого пришлось 16,9% продаж мороженого. Продажи по каменистой дороге увеличились вдвое, продажи бананов увеличились на 50%, а продажи тыквы — на 20%. Если у мороженого по каменистой дороге было на один процент меньше продаж, чем у бананового мороженого, узнайте, какой процент продаж мороженого было произведено каждым отдельным мороженым в прошлом году.

                            Банан — 3%, тыква — 7%, а каменистая дорога — 2%

                            Пакет с ореховой смесью содержит кешью, фисташки и миндаль.Всего в сумке 1000 орехов, а миндаля на 100 меньше, чем фисташек. Кешью весит 3 г, фисташки — 4 г, миндаль — 5 г. Если мешок весит 3,7 кг, узнайте, сколько орехов каждого вида в нем.

                            Пакет с ореховой смесью содержит кешью, фисташки и миндаль. Изначально в сумке было 900 орехов. Было съедено 30% миндаля, 20% кешью и 10% фисташек, и теперь в сумке осталось 770 орехов. Изначально кешью было на 100 штук больше, чем миндаля.Для начала выясните, сколько орехов каждого типа было в пакете.

                            100 миндальных орехов, 200 кешью, 600 фисташек

                            Глоссарий

                            дополненная матрица
                            матрица коэффициентов, примыкающая к столбцу констант, разделенному вертикальной линией в скобках матрицы.
                            матрица коэффициентов
                            матрица, содержащая только коэффициенты из системы уравнений
                            Исключение по Гауссу
                            с использованием элементарных операций со строками для получения матрицы в виде строки-эшелона
                            главная диагональ
                            записи из левого верхнего угла по диагонали в правый нижний угол квадратной матрицы
                            рядная форма
                            после выполнения строковых операций матричная форма, содержащая единицы по главной диагонали и нули в каждом пробеле ниже диагонали
                            эквивалент строки
                            две матрицы и эквивалентны по строкам, если одна может быть получена из другой путем выполнения основных операций со строками.
                            строковые операции
                            добавление одной строки к другой строке, умножение строки на константу, перестановка строк и т. Д. С целью получения формы «строка-эшелон»

                            Как использовать метод исключения Гаусса для решения системы уравнений?

                            ПРИМЕР:

                            Используйте метод исключения Гаусса для решения следующей системы уравнений.

                            # x + 2y + 3z = -7 #
                            # 2x-3y-5z = 9 #
                            # -6z-8y + z = -22 #

                            Раствор:

                            Настроить расширенную матрицу формы.

                            # ((1,2,3, |, -7), (2,3, -5, |, 9), (- 6, -8,1, |, 22)) #

                            Цель 1. Получите 1 в верхнем левом углу.

                            Уже сделано.

                            Цель 2a: Получите ноль под 1 в первом столбце.

                            Умножьте строку 1 на # -2 #, чтобы получить

                            # ((- 2, -4, -6, |, 14)) #

                            Добавьте результат в строку 2 и поместите результат в строку 2.

                            Обозначим операции как # -2R_2 + R_1 → R_2 #.

                            # ((1,2,3, |, -7), (2,3, -5, |, 9), (- 6, -8,1, |, 22)) stackrel (-2R_1 + R_2 → R_2) (→) ((1,2,3, |, -7), (0, -7, -11, |, 23), (- 6, -8,1, |, 22)) #

                            Цель 2b: Получите еще один ноль в первом столбце.

                            Для этого нам понадобится операция # 6R_1 + R_3 → R_3 #.

                            # ((1,2,3, |, -7), (0, -7, -11, |, 23), (- 6, -8,1, |, 22)) stackrel (6R_2 + R_3 → R_3) (→) ((1,2,3, |, -7), (0, -7, -11, |, 23), (0,4,19, |, -64)) #

                            Цель 2c. Получите оставшийся ноль.

                            Умножьте строку 2 на # -1 / 7 #.

                            # ((1,2,3, |, -7), (0, -7, -11, |, 23), (0,4,19, |, -64)) stackrel (- (1/7 ) R_2 → R_2) (→) ((1,2,3, |, -7), (0,1,11 / 7, |, -23 / 7), (0,4,19, |, -64 )) #

                            Теперь используйте операцию # -4R_2 + R_3 → R_3 #.

                            # ((1,2,3, |, -7), (0,1,11 / 7, |, -23 / 7), (0,4,19, |, -64)) stackrel (-4R_2 + R_3 → R_3) (→) ((1,2,3, |, -7), (0,1,11 / 7, |, -23 / 7), (0,0,89 / 7, |, -356/7)) #

                            Умножьте третью строку на # 7/89 #.

                            # ((1,2,3, |, -7), (0,1,11 / 7, |, -23 / 7), (0,0,89 / 7, |, -356 / 7)) stackrel (7 / 89R_3 → R_3) (→) ((1,2,3, |, -7), (0,1,11 / 7, |, -23 / 7), (0,0,1, | , -4)) #

                            Цель 3. Используйте обратную подстановку, чтобы получить значения # x #, # y # и # z #.

                            Цель 3а. Рассчитайте # z #.

                            #z = -4 #

                            Цель 3b. Рассчитайте # y #.

                            # y + 11 / 7z = -23 / 7 #
                            # y-44/7 = -23 / 7 #
                            # y = 44 / 7-23 / 7 = 21/7 #

                            # y = 3 #

                            Цель 3c. Вычислить x.

                            # x + 2y + 3z = -7 #
                            # x + 6-12 = -7 #
                            # x-6 = -7 #

                            # х = 1 #

                            Решение: # x = 1, y = 3, z = -4 #

                            Гауссовский метод исключения и матричных уравнений

                            Форма рядового эшелона

                            Часто при работе с расширенными матрицами мы понимаем, что одни матрицы более интересны, чем другие.Когда матрица имеет только нули под ее диагональными элементами, то говорят, что она находится в эшелоне формы или эшелоне строки формы .

                            Вот пример

                            Форма Echelon интересна тем, что позволяет нам более легко решить систему, используя технику, называемую , обратная подстановка . Предполагая, что вышеуказанная матрица является расширенной матрицей, тогда каждый из первых пяти столбцов представляет переменную, скажем, a, b, c, d, e .Это означает, что, посмотрев на последний столбец, мы сразу увидим, что 4 e = 1 , так что e = 1/4 . Теперь мы можем заменить e во второй предпоследней строке. Это дает нам d + 7e = 5 , так что d + 7/4 = 5 , так что d = 27/4 . Теперь мы можем подставить d и e в третью строку и так далее, пока не получим значения для всех пяти переменных.

                            Уменьшенная форма ступенчатого эшелона

                            Другая форма, аналогичная эшелонной, — это сокращенная форма эшелона .Для любой матрицы первая ненулевая запись в строке называется точкой поворота . Матрица находится в форме сокращенного эшелона строк, когда каждая точка поворота равна 1, а точка поворота является единственной ненулевой записью в ее столбце . Вот идеальный пример:

                            Если дана матрица в форме сокращенного эшелона строк, если в каждом столбце есть точка поворота, мы можем просто прочитать решения, посмотрев на последний столбец. Итак, если мы снова предположим, что первые пять столбцов соответствуют переменным a, b, c, d и e , тогда a = 3 , b = 1 , c = 2 , d = 5 и e = 2 .

                            Вскоре мы увидим технику, которая берет любую матрицу и переводит ее в сокращенную форму эшелона строк, тем самым решая любое уравнение или систему, которые она представляет. Вот еще один пример, который показывает, как решения соотносятся с сокращенной формой эшелона строк.

                            .

                          Добавить комментарий

                          Ваш адрес email не будет опубликован. Обязательные поля помечены *